Você está na página 1de 90

flaps/slats wet V1 speed

I was out flying with


some friends and was
asked, "what do you
think creates more lift,
flaps or leading edge
slats?" I responded
quickly with flaps, just
because of the camber
created by flaps.
What do you all think
creates more lift?
Also, how does a wet
V1 speed effect weight,
performance, etc.?
I told him that the V1
speed would be lower
and your weight would
be limited and finally
your obstacle clearance
would be degraded from
35ft to 15ft.
Does this sound right?
Thanks in advance!

31st October 2003, 08:02

HotDog
The Reverend
Join Date: Oct 1999
Location:
Sydney,NSW,Australia
Posts: 1,945

1st November 2003, 00:13

mutt
Join Date: Sep 1999
Location: ME
Posts: 3,622

#2 (permalink)

Lufthansa 747-130 crashed on T/O in Nairobi, 20 Nov. 1974 due to


the failure of the LE flaps to extend although TE flaps were in the
T/O range.
V1 is reduced by 10KTS and this will produce screen heights
between 15 - 25ft.

#3 (permalink)

You will find that a number of aircraft are approved to takeoff with
only leading edge slats extended, flaps zero. I dont know of any
that are approved with only flaps extended. This would lead me to
say that slats are more effective.
Wet V1 changes really depend on the aircraft date of
manufacture. 4 engine aircraft usually only have a speed
decrement, twin engined aircraft will have a speed and weight
decrement. The V-speed decrements can be a lot higher than the
10 knots mentioned above.

Mutt.

#4 (permalink)

1st November 2003, 22:04

Zoner
Join Date: May 2001
Location: Area 52
Posts: 169

A few years back I had to do a ferry flight out of Sao Paulo with
the LEDs retracted on a B-747-100. Even with low gross weight
and greatly increased takeoff speeds Boeing told us to expect
some buffeting after liftoff. They were right. I did many zero flap
takeoffs with slats extended on the DC-9-30 and everything was
pretty normal. So I'd have to say I would want the LEDs if I could
only choose one.

#5 (permalink)

2nd November 2003, 06:22

Tonic Please
Join Date: Jul 2002
Location: Paris, France.
Age: 25
Posts: 625

Reminds me of the 727 (i think) aircraft that crashed on departure


from Heathrow back in the 70s.(27L/R runways - it hit ashford or
somewhere west of airfield I know that) Dad was one of the first
people on the scene!!
The truth emerged about the slats (leading edge), being retracted
too soon, thus resulting in increased drag and reduced lift due to
decreased accel.
Anyone care to elaborate for me?
Dan

#6 (permalink)

2nd November 2003, 07:02

HotDog
The Reverend
Join Date: Oct 1999
Location:
Sydney,NSW,Australia
Posts: 1,945

I think you mean this one?


Heathrow, England, June 18 1972 - 118 killed
British European Airways HS-121 Trident 1C, crew failed to
diagnose premature Leading edge flap retraction.

#7 (permalink)

2nd November 2003, 10:48

EFP058
Join Date: Oct 2003
Location: Krautland
Posts: 49

Excuse my ignorance please, but what exactly do you mean by


the term "wet V1 speed"? Rainy weather?

#8 (permalink)

2nd November 2003, 12:08

HotDog

Wet runway surface V1.

The Reverend
Join Date: Oct 1999
Location:
Sydney,NSW,Australia
Posts: 1,945

#9 (permalink)

2nd November 2003, 13:58

Jagbag
Join Date: Jun 2002
Location: Northern
Hemisphere
Posts: 88

Slats or LE Flaps Vs TE Flaps

Slats improve the low speed handling of an aerofoil by a large


amount due to delayed separtion of boundary layer, increased lift
generating surface etc. while TE or LE Flaps increase the camber/
total surface area and thereby the lift and drag.
This becomes critical in a swept wing planform which has LE Flaps
near the wing tips where the BL separation is ahead due to the
transverse flow across the wing.
If the Angle of attack and the speeds which are meant for slats
extended are used when slats have failed- then the aircraft is very
close/in the region of reverse command, where there could be a
large rise in drag for a correspondingly reduced rise in lift. if this
is not detected quickly by a pilot then it could lead to a descend
because the lift equation of the aircraft is not matching up. Hence
the need to critical monitor airspeed and attitude during take
off/go around by BOTH pilots. Hence any failure either LE or TE
could result in disaster. Its happened to me when my co pilot
raised the flaps to 0 by mistake instead of flaps 15 during a single
engine (raw) go around at minima of an ILS. Luckily managed to
control it. thank god it was in the simulator (but during a test).
However my stick shaker did come on momentarily.
The situations enumerated by other contributors have been when
the slats have failed but one knew that he/she had no flaps. A
crash may happen even if the flaps fail. So beware of drawing half
conclusions.
Hope this helps.

#10 (permalink)

2nd November 2003, 16:41

Tonic Please

Hotdog

Join Date: Jul 2002


Location: Paris, France.

Spot on thats it. Thought id mention it to shed light on how

Age: 25
Posts: 625

serious LE and TE issues can get.


Smooth skies
Dan

#11 (permalink)

2nd November 2003, 17:42

aviate1138
Join Date: Jan 2003
Location: Surrey Hills
Posts: 925

Tonic Please said...


'Reminds me of the 727 (i think) aircraft that crashed on
departure from Heathrow back in the 70s.(27L/R runways - it hit
ashford or somewhere west of airfield I know that) Dad was one
of the first people on the scene!!'
Captain had heart attack just after takeoff. Confusion on the flight
deck led to the Trident deep stalling into a postage stamp field
near Staines by-pass and missed the 150,000 volt pylons by just
a few feet. Virtually no forward velocity and full fuel [ nearly] it
did not catch fire but the massive vertical G component killed
everyone. Sad thing was the BBC and Independant Radio had to
keep broadcasting on all their radio/TV channels for ghoolish
rubberneckers to get the f**k out of the way as Fire, Police and
Ambulances couldn't get to the scene! Then the souvenir hunters
moved in - some sick people were out there that week. Made me
sad to be a Brit for a while.
That one accident made the CAA paranoid about cardiac
assessment from then on.
Aviate 1138

#12 (permalink)

2nd November 2003, 18:05

Tonic Please
Join Date: Jul 2002
Location: Paris, France.
Age: 25
Posts: 625

Awful. Just like to make clear Dad didnt get any souvernirs. He
tried to help.
He was only in his teens and not particularly into
aviation.

#13 (permalink)

3rd November 2003, 12:13

EFP058
Join Date: Oct 2003
Location: Krautland
Posts: 49

Quote:

Wet runway surface V1.


Ah, okay, thanks. I thought it would be something much more
complicated, but I guess sometimes even in aviation things can be
simple.

Quote:

Lufthansa 747-130 crashed on T/O in Nairobi, 20 Nov. 1974 due


to the failure of the LE flaps to extend although TE flaps were in
the T/O range.
I believe the LE flaps didnt fail to extend but were rather
accidentally retracted again prior to t/o.
This is what my level of information is about this particular
accident: The 747 was taxiing to the active RWY, and due to the
taxiway having quite a bit of downslope the crew was using idle
reverse in order to avoid riding the brakes. Problem is, the 747
retracts the LE flaps when reverse thrust is applied. Apparently this
can be overridden somehow (sorry for the lack of details, but Im
not exactly an expert about the systems of a 747, Im sure
someone else will correct me if Im wrong), but for some reason
the crew neglected to do so.
Problem number 2 was that the warning light for the retracted LE
flaps was only present on the FEs panel, but he didnt notice the
warning since he was already facing forward for the departure.
So the crew started the t/o roll and rotated at the calculated Vr
speed. The aircraft actually lifted off, but instead of climbing out it
wobbled along a few feet above groundlevel (would it be possible
that it was flying only due to the ground effect?), and of course
since it lifted off without LE flaps it was extremely close to stall
speed. The crew was puzzled by the unexpected behavior of the
aircraft, and as a first measure they decided to get the gear up in
order to gain airspeed. However, when the gear bay doors opened,
that was all that was needed to cause enough increase in drag,
pushing back the airspeed just enough for the aircraft to stall.
The aircraft hit the ground about one kilometer behind the runway
and skidded for another half kilometer before it came to a rest and
burned out. A total of 59 people lost their lives in this accident, 4
crew and 55 pax.
The accident report claimed "the lack of warning of a critical
condition of leading edge flap position and the failure of the crew to
complete satisfactorily their checklist items" as major contributory
factors.
#14 (permalink)

3rd November 2003, 16:06

denachtenmai

Nairobi 747

Join Date: Aug 2001


Location: S.A.M. u.k.
Age: 67
Posts: 75

Ref the Lufthansa 747 crash. If my memory is correct, then I


think that a little while before this a similar situation occurred with
a BOAC 747, it did not result in an accident. I seem to recall that

the incident was reported to Boeing but the information was not
promulgated before the Nairobi crash. Maybe someone more
knowledgable than myself can add further comments.
Regards Den.

#15
(permalink)

3rd November 2003, 18:06

52049er
Join Date: May 1999
Location: Midlands
Posts: 184

Just for
completeness
a quote from
the AAIB
report
"The field was
sufficiently
inaccessible
to prevent all
but themost
persistent
sightseers
from reaching
it. The police
were
successfulin
controlling
spectators,
and
contemporary
reports that
membersof
the public had
impeded
rescue
services by
their presence
nearthe scene
are not borne
out by the
facts"
52

3rd November 2003, 19:20

HotDog
The Reverend

#16
(permalink)
Quote:

Join Date: Oct 1999


Location: Sydney,NSW,Australia
Posts: 1,945

The 747
was taxiing
to the
active RWY,
and due to
the taxiway
having
quite a bit
of
downslope
the crew
was using
idle reverse
in order to
avoid riding
the brakes.
Problem is,
the 747
retracts the
LE flaps
when
reverse
thrust is
applied.
Apparently
this can be
overridden
somehow
(sorry for
the lack of
details, but
Im not
exactly an
expert
about the
systems of
a 747, Im
sure
someone
else will
correct me
if Im
wrong), but
for some
reason the
crew
neglected
to do so.
The leading
edge devices
take 7 seconds

to extend with
T/O flaps
selected so
they would
have been well
and truly out
long before VR.
The East
African
Community
Accident report
stated the
cause of the
accident was
the failure to
select the
pneumatic
power source
for LE device
extension
Question about turboprops

I'm about to start the transition course from the venerable Bonanza A36 to
the Piper Cheyenne or the Beech King Air.
Although I've studied a lot about jet aircrafts and it's systems, I know little
or almost nothing about turbopropellers.
The one thing that intrigues me the most is the pressurization controls at the
King Air, as I've seen in pictures. It's seems to lack any automation at all!
It's worst than the 737-200 pressuruzation controls, even the B200 or the
B350.
How this system works and why it still made this old-fashioned way?
Thank's for any responses!

#2
(permalink)

5th November 2003, 17:55

OzExpat
PPRuNeaholic

Join Date: Jun 2000


Location: Port Morbid

If I were to
invoke haiku
for this, I'd
say that it is
because it is.
And that's
about the
whole story. I

Posts: 3,257

don't know
how old you
think Kingairs
are, but the
fact is that its'
pressurisation
system was,
like the
aeroplane
itself, not
made
yesterday.
Personally,
I've never
had a
problem with
the
functionality
of the system
and really fail
to understand
why this
seems such a
problem. I set
the
pressurisation
control to
where I want
it and monitor
cabin climb or
descent, and
the
differential, to
make sure
everything
happens the
way it should.
Once I've set
it, the system
actually IS
automatic in
that it will
take the cabin
to the desired
cabin altitude
and maintain
it for me.
All I have to
do is watch to
make sure it

happens.
And, of
course, I
need to know
what to do
about it if it
doesn't
happen.

#3 (permalink)

5th November 2003, 20:08

E1453
Join Date: Nov 2003
Location: Brazil
Posts: 52

Thanks!

Thank's for you response.


The fact is I've never been on a King Air or Cheyenne, and I don't
have the manuals yet.
Looking at the pictures of the King's cockpit, I saw the
pressurization panel I found it odd. I'm so used to the Boeing
737's systems (wich I learned during my graduation) that I
thought the King's system odd.
Otherwise, could anyone explain me what is King's or Cheyenne's
performance capabilities in case of a engine failure on takeoff?
I'm know the FAR Chapter 25's rules, but never saw the
requirements for turboprops in general aviation.
Thank's!

#4 (permalink)

6th November 2003, 03:41

vunzke

Hi E1453,

Join Date: Nov 2003


Location: Netherlands
Posts: 34

Performance wise the King Air is a beautifull plane as it is in every


other aspect. I've done about 2000 hrs on it in VIP charter and
loved the plane. Single engine performance is great and with the
rudder assist (also a very basic but non the less effective system)
a very pilot friendly aircraft.
Id say, sit back and relax, listen to you instructor and study the
books and youll enjoy the ride from the first flight on...
have fun

6th November 2003, 18:09

#5 (permalink)

OzExpat
PPRuNeaholic

Join Date: Jun 2000


Location: Port Morbid
Posts: 3,257

Yeah, I've got a few thousand hours on Kingairs, C-90

and B200

The latter is what I refer to as a gentleman's aeroplane. It is a


very versatile aeroplane indeed with good slow speed
performance that's got me out of a few difficult situations. In one
instance, due to an ATC stuff-up, I ended up at 10,000 feet at 10
DME on approach and was able to rescue the situation with a 1:1
descent profile very nicely.
I have an idea that, after a particularly nasty prang on take-off
from Sydney, involving a B200, many years ago some flight tests
were conducted. If my aging memory serves me correctly, the
tests found that. at MTOW in given conditions (and with a pilot
who was ready for it), an engine failure prior to V1 could be
negotiated without rejecting the take-off. That is, the aeroplane
actually accelerated to V1 on one engine and got airborne, then
flew a circuit and landed again.
Clearly, not the sort of thing to explore too closely, but an
indicatuon of just how good the aeroplane really is.
I've enjoyed all my B200 flying immensely and can recommend it
to you very highly. As vunzke says, "sit back and relax, listen to
you instructor and study the books and youll enjoy the ride from
the first flight on". I'm sure that you'll enjoy this very forgiving
and flexible aeroplane.

#6 (permalink)

6th November 2003, 18:39

E1453

V1, second segment, etc

Join Date: Nov 2003


Location: Brazil
Posts: 52

Do the King Air have a published V1, VR and V2 for each weigh &
temp?
In case of a engine failure at V1, will the King overfly the runway
end at or above 35 ft?
What V1, VR, V2 and VREF is common to use?
Can you use reduced thrust (or assumed temperature) on
takeoff?
Any problem in reducind prop rpm first, before reducing the
torque, at the first reduction after takeoff?
Sorry for asking so many questions, but thank's for your help.

7th November 2003, 09:10

604guy
Join Date: Jun 2003
Location: Canada
Age: 55
Posts: 195

#7 (permalink)

Pressurization controls on all Kingairs (except perhaps for some


early A90's) are extremely easy to operate. The same can be said
for Cheyennes again except for some early II's. In the early
models of those you have to select cabin altitude and then after
takeoff select ascend, neutral at top of climb and then when ready
for descent, reset cabin altitude and select descend. Not likely
you'll run into that unless you get into something manufactured
mid70's vintage.
There is not really anything to gain in using reduced thrust
takeoffs in this catagory of aircraft and might get you into a bad
situation one day if Mr Murphy raises his head and you have an
engine loss as the gear is going in the wells.
If you reduce prop RPM first after takeoff without reducing torque
first you will almost certainly exceed torque limlts. Torque will
increase as you reduce prop RPM.

7th November 2003, 14:02

#8 (permalink)

compressor stall
Join Date: Feb 2000
Location: 500 miles from
Chaikhosi, Yogistan
Posts: 1,920

From the 200/200C POH. Note these are not Raisbeck modified
KingAirs!
When looking at the figures remember that you are dealing with
an FAR23 certified aeroplane. In a nutshell all it is required to do
is maintain a positive rate of climb on one engine up to a certain
height.
Beechcraft offer the computations and charts for a balanced field
length. They state that it is possible to reduce weight to "obtain
the performance specifications of FAR 25 during the critical takeoff
and initial climb segments. ...full compliance with other
regulations applicable to FAR25 is not implied." It is quite an
involved process and a lot of iterative calculations need to be
performed.
(the example they use is on a hot day at 5000' field, 2+kms of
rwy and 10 kts headwind, you need to limit your weight by 2000
lbs below MTOW!)
So the following answers deal specifically with non FAR 25 charts
Do the King Air have a published V1, VR and V2 for each weigh &
temp?
They publish a rotate speed and a 50 foot speed for different
weights (Vr is always 95kias and 50' is 121@12500 down to
108kias@9000lbs.
V1 equals Vr according to the notes.

In case of a engine failure at V1, will the King overfly the runway
end at or above 35 ft? Refer the charts and the comments above.
If light yes, it should.
What V1, VR, V2 and VREF is common to use?
See above figues. Approach Speed (they don't use the term Vref)
is 103kias@12500lbs down to 93@9000.
Can you use reduced thrust (or assumed temperature) on
takeoff?
Why would you want to make a performance that is marginal
even more so?
Any problem in reducind prop rpm first, before reducing the
torque, at the first reduction after takeoff?
Yes - your torque will go though the limits. Takeoff torque @
2230ftlbs . RPM @ 2000. we reduce to 2000ftlbs and 1900 at first
power reduction (LSALT at night/IMC). To easiest procedure is
reduce the power to 1900ftlbs and then pull the props back to
1900rpm. The torque will rise to about 2000ftlbs then and only
small change necessary to peg it exactly.
A lovely machine to fly. A tad thirsty perhaps, but a new Kingair
climbs out like a homesick angel. As for the pressurisation - it's
pretty simple really. Before t/off set the pressurisation controller
to the cruise alt plus 1000 and before TOD set the airfield
elevation plus 500' (there is a chart to be more specific if you
must). Wind it slowly to begin with to avoid cabin drop, but it
works fine. Smoother than the PC12's pressurisation controller!

#9 (permalink)

7th November 2003, 19:09

E1453
Join Date: Nov 2003
Location: Brazil
Posts: 52

"power" reduction a S/E performance

Sorry for asking, asking, asking, again and again!


Does the manual say the King will fly, after an engine failure at
rotation, at standard day, with MTOW?
Is there a takeoff chart with the Performance-Limited MTOW for
runway lenght, slope, temp & elevation?
What the FAR23 says about single engine performance on takeoff
for King-like planes?
I though it would be interesting to reduce temps with a reduced
power takeoff, to save the hot section of the engines. If there
were takeoff charts like the ones on Boeing's manuals, with field
limited and climb limited performances, we would be able to

calculate engine power to meet at least FAR 23 requirements,


when not fully loaded.
What do you think?

#10 (permalink)

8th November 2003, 00:01

OzExpat
PPRuNeaholic

Join Date: Jun 2000


Location: Port Morbid
Posts: 3,257

I suspect that you might have misinterpreted my previous post.


The B200 Kingair is not a FAR25 aeroplane. Therefore you cannot
think about it in those terms. The incident to which I referred in
my previous post occurred because, it is alleged, that the pilot
used a reduced thrust take-off technique for noise abatement
purposes. It is generally assumed that he believed that there was
sufficient runway length for that and, indeed, that was probably
true... if and only IF nothing went wrong during the take-off.
Well, something went horribly wrong.
It is never a good idea to consider reduced thrust in any
aeroplane of that class. There is nothing to be gained by it as the
noise footprint is far less, even at max take-off thrust, than the
average jet. This, of course, is simply my own opinion, but I feel
sure it is a widely held opinion.
There ARE performance limitations but, for take-off, they mostly
resolve into the clash between torque and temperature (ITT). At
least, that's the case in my experience. We have SOPs that say
that, if we have 1000 metres of runway up to about 1,000 feet
elevation, we can go at MTOW and still meet balanced field length
considerations.
I operate in the tropics where the average temperature is
between ISA +15 and ISA +20, so an elevation of 1,000 feet can
produce a density altitude of around 2,500 feet. I operate into
aerodromes up to 5500 feet AMSL (some operators take B200s in
and out of airstrips at much elevations) but we have SOPs to deal
with the reduced take-off weight.
The performance limitations based on runway length and
associated environmental conditions are related to take-off
weight. These charts always assume that you will use the ITT and
torque associated with the conditions. To that extent, yes, there is
an argument concerning reduced take-off thrust, but I think you
need to read the Manual to understand it.
The B200 is not a Boeing. I can do no more than to strongly
recommend that you study the Manual, believe it, but add a fudge
factor to ensure that you can enjoy a beer at the Aero Club after

each flight.
It is a great aeroplane and a very forgiving one, but it DOES have
its limitations, just like every other aeroplane. If you follow all the
good advice that has been offerred so far in this thread, by all
those who have replied, you'll enjoy flying the aeroplane. Just,
please, please, please, consider it as more of a GA type aeroplane
than a Boeing.
In fact, if you forget all about the Boeing, you might be better off.
That way, you can explore its' performance gently, in a more
controlled and professional manner, and find out for yourself what
it will do for you when the chips are down.
The post by my good mate compressor stall is spot-on, so I
recommend that you take a copy of it for reference in your
training.

#11 (permalink)

8th November 2003, 09:02

compressor stall

You might want to study these links.

Join Date: Feb 2000


Location: 500 miles from
Chaikhosi, Yogistan
Posts: 1,920

FAR 25
FAR 23
Of course just open and print the relevant bits. Place them side by
side and understand what FAR 23 does NOT say.
CS
G'Day OzEx what's to go with the geography movement? you got
a new job or sumfink?

#12 (permalink)

8th November 2003, 12:30

E1453
Join Date: Nov 2003
Location: Brazil
Posts: 52

FAR 23 compliance

I read the FAR23, about commuter planes. Very enlightening.


Well, please correct me if I'm wrong, but I think the T/O
performance for a King-like (commuter) turboprop is:
T/O runway requirements:
2 engines running: 115% of the distance required to climb to 35ft
at not less than V2
1 engine running: the distance required to climb to 35ft, at not

less than V2, in the case of a failure after V1, of course


climb requirements:
gear down, runway level: must demonstrate a measurable climb,
T/O power & T/O flaps, not touching the engine levers
gear up: 2% climb gradient up to 400ft, T/O power, T/O flaps, not
touching engine levers
above 400ft up tp 1500ft: 1,2% climb gradient, MCP, flaps up
Well, thanks for the advice, compressor stall. It did help a lot.
I can now figure out that the capabilities of commuters are not
comparable to those of transport jets. It's impossible to comply
with any SID's climb gradients. For me, the most critical moments
would be beetween the liftoff and the gear retraction. I can
imagine the consequences of a propeller failing to autofeather.

I forgot to mention:
Knowing the marginal performance beetween liftoff and V2, I
know realise why you folks don't recommend any power reduction
at T/O. Before retracting the gear, it may be hard to keed this
baby flying! Every single horse would help a lot in such a
situation!
Thank's!

8th November 2003, 20:32

Tinstaafl
Join Date: Dec 1998
Location: Escapee from
Ultima Thule
Posts: 2,930

#13 (permalink)

Why do light twins , including commuters, have two engines?


Because they need BOTH of them.
At best they're the equivalent of a single engine a/c with half its
engine mounted 'over there' and the other half mounted 'over
here'. This means that an engine failure is constrained to be more
like a 'partial' failure of the (single) engine.
Like single, a partial failure may leave you with enough power to
maintain flight......or it may not. Instead of having sufficient
excess of power to guarantee performance (two engine or FAR25
a/c) you're now in the situation where you just barely have
enough **IF** the a/c is configured optimally. BTW, gear down,
flap down & prop windmilling is not 'optimal'....

#14 (permalink)

8th November 2003, 22:45

OzExpat
PPRuNeaholic

Join Date: Jun 2000


Location: Port Morbid
Posts: 3,257

9th November 2003, 00:43

E1453
Join Date: Nov 2003
Location: Brazil
Posts: 52

I'm glad that there's been enough help for you, E1453, in
understanding what the aeroplane is certified to do. Having a
healthy respect for it will help to ensure that you enjoy the time
you spend flying it.
stallie... no such luck mate. It's just a two-month swan on
expenses! Nothing less than I deserve, of course!

#15 (permalink)
engine
management

I have a few
more questions:
I know that the
RPM has an
effect on torque,
just like the real
engines (the
reciprocrating big
bores are more
fun!), but, is
there any effect
of RPM on TIT?
What's the best
condition for
cruise, in terms
of engine
management? It
includes: fuel
efficiency, engine
longevity (low
temps) and
noise.
I never saw an
turboprop power
chart, but I
would like to
know the best
combination of
torque x RPM to
achieve the

above results.
Surely RPM's
leads to low
noise levels, but
is it good to fuel
efficiency? Is it
good for engine
longevity?
Does the use of
de-ice devices
reduce the
engine power
(propeller output,
to be more
accurate)
significantly?
Thank's in
advance!
I have a few
more questions:
I know that the
RPM has an
effect on torque,
just like the real
engines (the
reciprocrating big
bores are more
fun!), but, is
there any effect
of RPM on TIT?
What's the best
condition for
cruise, in terms
of engine
management? It
includes: fuel
efficiency, engine
longevity (low
temps) and
noise.
I never saw an
turboprop power
chart, but I
would like to
know the best
combination of

torque x RPM to
achieve the
above results.
Surely low RPMs
leads to low
noise levels, but
is it good to fuel
efficiency? Is it
good for engine
longevity?
Does the use of
de-ice devices
reduce the
engine power
(propeller output,
to be more
accurate)
significantly?
Thank's in
advance!

9th November 2003, 07:05

Sheep Guts
Props are for boats!
Join Date: Oct 2000
Location: An Asian Hub
Age: 42
Posts: 875

#16 (permalink)

Gday E1453,
Well good posts
sofar all the B200
Gurus(OzExpat
and Stallie). Im
not one,maybe a
A90 Guru and a
Twotter hacker.
But mate
generally engine
wear with
Turbines is done
during start up
and shut down.
You have to take
note of the Limits
in ITT, and stick
to them, but not
use them as a
power setting (as
some people).
For example on a
A90 the Max
continous ITT in
the cruise is 705.
Now thats a

limit , never set


it in the cruise.
Also monitor and
record you start
temps. The
limitation we
know for all PT6s
is 1090 for 2secs.
And not to add
fuel unless you
have a sustained
N1 RPM of 12 %.
Well ill tell you it
will start at 12%!
But man your
gonna get a
really exciting
start temp from
that. The trick is
to recognise this
before adding
fuel, could be
bad GPU, Battery
or
Startergenerator.
Youll here it spin
up alot weaker
and you may
notice the Bat
volts get loaded
down t0o much.
If it is abort the
start and get a
tech to check it
out , dont add
fuel, or you get a
start , but you
may shave many
hours off a good
engine. Every
engine temp
spike contributes
to blade and Hot
section damage
over the long
term.
Funny thing,
people talk about
reduced power
take offs in Turbo

Props. A
definition of this,
maybe needed.
Basically we all
do reduced
power takeoffs,
especially in the
Tropics because
we get to Our ITT
limit much more
easily, due to the
increased
ambeint Temp.
When I I flew
Twotter -200S
WITH -20
engines we had
90% power
takeoff cards that
we used to set
the TO Torque
with OAT and
TOW, if I am
recalling basically
it was the same
input as on the
Torque computer
which Dehaviland
supply with all
their Twotters.
We would have
reduced torque
settings. If there
would be a
setting for 2
engines and one
for 1INOP which
was alittle higher.
Ofcourse when I
moved to
-300Twotter the
temps limits
were still there
but the engines
were alot more
accomadating.
Yes bleed air
effects power

output. But only


slightly. With
nothing touched
after turning on
bleed air you
notice a slight
reduction in
torque and slight
increase in
FFlow.
Fuel efficeincy
you will get
better with
altitude ,just like
your 737, no
difference really.
And the most
optimum will be
very close to
your Max ITT
Cruise limit
incidently.
When I level off
at around FL200
in my A90, I set
up the props to
1900 RPM as
specified. But if I
forget to bring
back the RPM to
1900 from 2000
then Ill notice
small diffference
in Torque and
Fuel flows. AT
2000 RPM for a
given torque
setting I get a a
higher fflow than
for say 1900RPM.
Why you ask?
Well its designed
for it the
Reduction Gear
Box and the
mutlitude of test
flight hours done
by the Beech
People got the
optimum setting.

All the Crusie


Power Tables in
A90 Manual
anyway use
1900PRM.
Anyway enough
of my blithering
Hope it helps
Sheep
Yep Ex A90s and
C90s are , but
hey thats what
all the other ones
got their genes
from
Engine Fire on Take Off. TOGA application.

What are your views on the application of TOGA during a FLEX T/O with
engine fire?
If the aircraft is a large twin jet and the fire bell goes off at V1, do you
consider it wise or not to apply TOGA on both (assuming the a/c is
climbing OK anyhow on FLEX power).

30th January 2004, 18:36

Spearing Britney
Join Date: Jan 2000
Location: Hang on, I'll check my roster...
Posts: 197

#2 (permalink)

If it goes at V1
then TOGA,
because you
simply don't
know how it will
climb - e.g. is the
wing damaged
too? If it goes
once airborne
and climb
continues
strongly then
remain at FLEX
but if in any
doubt then apply
TOGA and risk
inducing failure
in an already

soon-to-beshutdown engine
for that extra few
feet that may
make all the
difference...
Too simplistic?

31st January 2004, 10:59

Elliot Moose
Join Date: Feb 2000
Location: Montreal
Posts: 116

#3 (permalink)

My understanding is that TOGA is not required. The whole point of


runway analysis and the computation of flex thrust is to ensure
that the aircraft will still meet all nominal climb requirements,
including engine out, the same as it would if the takeoff were
actually executed at the flex temperature.
That said, if one has presence of mind to apply TOGA thrust, then
so much the better, as you would perform with that much more
"comfort room" and, as stated above it would certainly be a good
idea in the event of severe structural damage. On at least some
(if not all) FADEC equipped aircraft, TOGA or APR is automatically
applied once the FADEC detects the single engine schedule.

31st January 2004, 22:57

Idunno

#4 (permalink)

I agree with both points, but aren't you concerned about the idea
of pumping 'X' kilos more fuel per second into a burning engine?

Join Date: Aug 2001


Location: Mid Atlantic
Posts: 478

31st January 2004, 23:25

#5 (permalink)

alf5071h

Leave the thrust alone, just fly the aircraft. Dont add more
complications; the thrust increase will change the rudder
Join Date: Jul 2003
requirement, which may feed into roll. Also dont start introducing
Location: An Island Province what ifs, the certification rules are such that an engine fire or
Posts: 697
malfunction is extremely unlikely to damage the wing. History /
accident statistics show that we are in far more danger from a
poorly flown climb out or a distraction of our own making than for
the lack of thrust.
Why get over concerned about pumping 'X' kilos more fuel per
second into a burning engine; you (we the industry) are quite
happy about climbing to a safe height before completing shut
down drills. For those who climb to 1000 ft aal, when on the WAT

limit (200 ft/min) thats 5 min of burning time, which is within


most design assumptions and safe.

31st January 2004, 23:41

FullWings
Join Date: Dec 2003
Location: Tring, UK
Posts: 503

#6 (permalink)

Having flown different types, some requiring TOGA thrust after an


engine 'failure', some not, I would say there are arguments in
both directions.
In the lower powered aircraft, it seemed like a good idea - get
away from the ground because you'll soon be on one less. In
some of the later turbofan twins, you were never short of power
and full thrust could actually make handling more difficult.
Scenario: Light A/C, full derate (flex). V1/Vr probably Vmcg/Vmca
limited. Adding asymmetric full thrust will put you near the edge
of the controllability envelope for what gain? If you are fully
derated there is ample fat in the system. I appreciate that the
original question referred to a 'fire' as opposed to loss of thrust
but a very high rate of climb is not always the best thing to have,
especially if there is a low level off. There are some nasty little
traps set for you if you end up in one of the 'altitude capture'
APFD submodes which manufacturers love to include on their
latest and greatest. These can be coped with but remember you
have probably only just put an A/P in and are now concentrating
on shutting down the 'right' engine, initiating an emergency turn,
etc. I believe an Airbus (A340?) was lost during testing in similar
circumstances.
To summarise: If TOGA thrust is a SOP, then follow it. If it is
discretionary then I would advise caution at low speeds/weights.
If you are not happy with the aircraft performance then add
power until you are...
Aside: Some years ago the company I work for upgraded to a
turbofan variant of a particular A/C but carried over most of the
SOPs from the older type, including mandatory TOGA power.
About a year later I heard that that at very low weights the FMC
V-speeds could end up close, possibly below Vmca (full thrust)
due to the FMC recalculating a new Vmca based on the derate. I
don't know the truth but it was an interesting proposition: lo and
behold suddenly new SOPs came out with minimum V1s and the
removal of mandatory TOGA application...

1st February 2004, 20:26

320DRIVER
Join Date: Jan 1999
Location: Europe

#7 (permalink)

I don't have any experience on derated takeoffs since we only use


the flex option. It is my understanding however, that once you
select a derated takeoff (as opposed to flex) you will only get the

Posts: 324

maximum derated power from the engine when you go to TOGA,


rather than achieving the engine's full TOGA.
This is due to the arguments raised above as regards Vmca etc.

#8 (permalink)

1st February 2004, 20:43

Hudson
Posts: n/a

WAT limit of 200 feet per minute rate of climb? Are you sure of
that? Methinks that is one big problem if you are talking about a
twin jet climbing at 150 knots V2 one engine inoperative at 2.4%
gradient of climb at instant of gear up.

#9 (permalink)

1st February 2004, 22:54

777Efoh

320driver,

Join Date: Jul 2003


Location: Hotels
Posts: 53

Don't know about the Airbus, but in the Boeing, pushing the TOGA
button removes the derate ie, full TO will be achieved.

1st February 2004, 23:02

320DRIVER
Join Date: Jan 1999
Location: Europe
Posts: 324

#10 (permalink)

In case of the assumed temperature method (Boeing) or Flex


method (Airbus) I agree that full TO would be available on moving
the thrust lever to the TOGA detent.
However, the derating concept is something different since you
are derating the engine for that takeoff.
There is some info on http://www.b737.org.uk/assumedtemp.htm
which could maybe explain the concept better than my posts.
Having said that, I cannot comment on the B777 but I would have
assumed that the concept is the same.

3rd February 2004, 12:35

typhoonpilot
Join Date: Nov 2001
Location: Middle East
Posts: 430

#11 (permalink)

The Boeing FCOM at my airline does indeed say that the first push
of TOGA after takeoff will, among other things, remove the thrust
"derates". The confusion lies in that at my airline we only use the
Assumed Temperature method so that is all that will be removed.
It is always best to think of true derating as hanging a different
size engine on the airplane, you can't very well go out and change
that engine once airborne, at least not very easily. That differs
from the AT method where you are just reducing the thrust
setting. It is quite easy to push the thrust levers forward to
recoup that reduction if necessary.

To add my two cents to the original question, no way would I add


power for an engine fire indication. Engine fire does not equal loss
of thrust. If anything you are struggling to get the nose down to
level off at the MFRA with an engine fire, why exacerbate the
problem with more thrust ?
Typhoonpilot

#12 (permalink)

5th February 2004, 02:54

m&v
Join Date: Dec 1999
Location: delta.bc.canada
Posts: 259

Flex,is the same as derated power,just another Co's name for it.
Derated thrust meets the 'cert' criteria for the 2.4%gradient on
ONE.The additional thrust at TOGA was a YAW consideration on
the 747(wait until airborn).having said that some Co's call for
TOGA thrust(on twins),every little bit helps, after takeoff.
It has been said that since the advent of 'reduced'thrust,in the
60's,the trend for full thrust when needed has been trained out of
the knowledge loop(natural common sense).Ergo aircraft still
flutter out o'the sky when a good handfull to the firewall would
have helped.....
2.4% climb gradient=@150knts=2.5miles@min.
2.4% mile =144'/mile@2.5miles=360'/minute rate o'climb

#13 (permalink)

5th February 2004, 04:29

alf5071h

Hudson, m&v, not all of us have the luxury of the high speed
V2s. Where large flap angles are used for runway performance
Join Date: Jul 2003
the climb speeds are low; thus 2.4 % at 110 Kts V2 and say a
Location: An Island Province ground speed of 100 Kts, only gives approx 200 ft/min.
Posts: 697

When considering SOPs, do we seek altitude for safety or time to


asses the situation? More thrust should give more alt but less
time at the critical period, and possibly at the expense of control
with risk of error. There is probably no set answer, thus each to
his own, but at least know why the SOP is written for a particular
operating method and not the other.

#14 (permalink)

5th February 2004, 20:53

Idunno

Thank you all for your responses.

Join Date: Aug 2001


Location: Mid Atlantic
Posts: 478

The reason I asked was that an instructor in my last sim ride was
teaching TOGA as an SOP in all engine fail scenarios.

Its not written as an SOP in our books (its for 'consideration by


the pilot') but he was utterly adamant that it should always be
set.
I'll continue to 'consider it' first I think.

7th February 2004, 02:42

m&v
Join Date: Dec 1999
Location: delta.bc.canada
Posts: 259

10th February 2004, 02:32

SPEEDBIRD5FP
Join Date: Aug 2001
Location: HERE AND THERE
Posts: 41

#15 (permalink)

idunno,what does
your/the flight
Manual say as to the
application of
'thrust'with a
fire/failure at
takeoff.....Remember
reduced /flex thrust a
luxury of generous
runway lengths(room
to spare -ease up on
the thrust).Once one
has a failure,use
what power you have
available(KISS)
Cheers

#16 (permalink)

Just a thought.
But on the boeing i
fly, granted it is quite
an old one (737-200,
SP177 autopilot), on
take off, u press the
toga and once you
get the throttle hold
at circa 63 kts, it
does'nt matter if you
press the toga again.
The throttles are in
effect, in arm mode,
where u can move
them manually, but
they will not move
automatically. The
throttles can be
repositioned to any
thrust setting, but
only when u have
throttle hold

annunication. It is in
effect the same as
doing a level change
descent. The
throttles come back
to idle, then they go
into arm mode and u
can set them
wherever u want.
On a go around
however, u press
toga and u get x
amount of thrust,
press toga a second
time and u get max
go around thrust.
It may be different
on this type, age of
the aircraft and the
engines being set by
epr's, which must be
set by 60 kts.
But as i said, just a
thought.
Speedbird.
Heavy jet operation re: V2

Any heavy jet pilots out there, any thoughts?... I'm trying to et my
head around the complexities of V-speeds!!
If after normal rotation on a normal (4 engine) heavy weight take off
the aircraft has not reached V2 would you maintain a slightly lower
nose attitude to increase speed to V2, or continue rotation and fly
below V2 and count on acceleration in the climb?
Would you consider increasing thrust if it were available?
Is flying below V2 on initial climb out unsafe?
Could you assume that the aircraft loading data was probably
incorrect and that the aircraft was actually heavier than what the
figures had been calculated for?
Does this kind of underperformance occur regularly?
Thanks.
Last edited by backin5 : 22nd October 2003 at 03:09.

22nd October 2003, 06:33

#2 (permalink)

Basil
Join Date: Jun 2001
Location: UK
Posts: 1,386

Slightly lower nose


attitude to
accellerate to V2 or
V2 +10kn. If that
doesn't work then it's
time to use up some
of that engine life
that engineers keep
going on about (only
joking, F.E.s)
If the aircraft has all
engines running and
does not achieve V2
following a normal
rotation then
something's wrong:
windshear, heavier
than planned, power
down, gear left down
Yes, it's unsafe
inasmuch as flight
tested margins are
being eroded and, in
the event of an
engine failure, you
could lose further
speed and be in a
high drag situation,
closer to Vs, stall in a
turn and also be
approaching Vmcg
I've usually found
that sort of thing's
caused by windshear
or to a lesser extent
immediately after
take-off, temperature
inversion which tends
to get you a little
later (another good
reason to achieve V2
+10)
Not regularly but at
max RTOW it
requires a bit of
attention but then so
do lightweight

departures zooming
(good tabloid term)
up to/through initial
cleared altitude.
The standard phrase
for drawing other
crewmembers
attention to any of
the foregoing is:
"What the F V K K 's
up wi' this thing!"

#3 (permalink)

22nd October 2003, 12:29

411A
Join Date: Mar 2000
Location: Arizona USA
Posts: 7,508

This was more of a problem with the old straight-pipe (non-fan)


powered heavy jet aircraft, ie: 707-321 (Pratt&Whitney JT4A) or
Conway powered machines.
After rolling 11,500 feet on a 12,000 foot runway, then climbing
at V2/400 fpm, one began to wonder....where is 'd thrust?
All the while hoping one didn't flame out.
With fan powered aircraft, much better runway performance is the
norm....usually

#4 (permalink)

23rd October 2003, 01:32

backin5

Thanks so much guys, I really appreciate the "heads up".

Join Date: Oct 2003


Location: australia
Posts: 12

In regard to legalities... is V2 the actual REQUIRED min. climb


speed? ie. 1.15VS? (I can't remember the actual figure), or is it
regarded as generally acceptable to operate below V2 if the
aircraft requires a 12.5 degree nose-up pitch attitude for the
initial climb-out?
What I'm getting at is: If the aircraft operating manual states a
12.5 deg pitch angle for initial climb, would you maintain this
pitch angle to get away from the ground even if the aircraft is
operating below V2?
Or would it be acceptable/ preferable to, say, fly 10 deg nose-up
pitch attitude and fly V2?
Thanks again for your help, it's really appreciated!

#5 (permalink)

23rd October 2003, 02:20

Hawker-rider

Allright,

Join Date: Nov 2000


Location: USA
Posts: 30

when an airplane's engine flames out for whatever reason and


continues the take-off, after rotation, passing through 35feet the
speed at that moment is called V2 or takeoffsafety speed.
So if you use standard practices as far as rotation etc is concerned
you will reach V2 right after rotation. at least the certificated V2
speed. So going below V2 is basically only possible to do when
you use practices that are non standard and not approved.
Offcourse there is such a thing a an "optimized V2" which is
approx 1.5Vs as opposed to the normal certificated V2 which is
1.2Vs
Remember V2 doesn't have anything to do with the speeds Vx
and Vy

#6 (permalink)

23rd October 2003, 02:31

backin5

...woops, I just realised how dumb I just sounded!

Join Date: Oct 2003


Location: australia
Posts: 12

I understand the definition (I think), but what I really want to


know is how an experienced pilot would fly a take-off if the
aircraft is not performing as well as it should for whatever
reason...
Is it more important to climb out at V2, but with a lower nose
attitude, or would you climb out at the ops manual specified nose
up attitude but fly below V2?
Is V2 a legal requirement purely for performance calculations (ie
for RTOW & CL Grad calculations), or a real flying performance
speed?
Thanks, any help??!!!
Last edited by backin5 : 23rd October 2003 at 02:59.

#7 (permalink)

23rd October 2003, 06:42

Hawker-rider
Join Date: Nov 2000
Location: USA
Posts: 30

23rd October 2003, 07:20

edited the post, because the comments I was referring too have
been removed, so my post wasn't making sense anymore
Last edited by Hawker-rider : 23rd October 2003 at 11:48.

#8 (permalink)

FE Hoppy
Join Date: Sep 1998
Location: Zurich
Age: 41
Posts: 802

Vmu is above Vr.


vr is the speed at which the pilot initiates rotation. It may not be
less than V1 or 1.05Vmca
Vmu it the speed at or above which the aircraft can safely lift of
the ground and continue the take off with out hazard.
V2 is the speed atained at 35feet OEI when the aircraft is rotated
at vr
it is selected to be
=>1.1vmca
=>1.2vs(1.13vs-1g)
=>vr
Yes you will be at v2 at 35 feet because your take off was planned
that way. if you are limited then the screen height will be at the
end of toda if not it will be somewhere before.
You must rotate the aircraft as per the SOP otherwise you
copramise the height atte end of toda. If the speeds don't match
then the take off calc was wrong.

#9 (permalink)

23rd October 2003, 11:51

Hawker-rider

???

Join Date: Nov 2000


Location: USA
Posts: 30

The airplane I currently fly has a Vmu below the rotationspeed per
the flight manual.
I agree with you that the Vmu ( is a fixed speed) is higher then
the LOWEST POSSIBLE Vr, but that that low of a Vr is not used in
operational speeds, because of the tailstrike that occurs.
Therefore the Vr in the books, for a balanced field length or
whatever isn't necessarily lower then Vmu.
Back to your main question though.
Mostly you rotate initially to the pitch attitude that is taught, and
proven in operation. Then you expect to get a certain acceleration
from that, both engines you will accelerate through V2, and then
gradually increase your pitch attitude ( or not) to climbout etc. So
if for some reason the speed doesn't increase as anticipated, (or
over rotated) I'd slowly decrease the pitch attitude to get the
speed increase. ( or should I say instead of decrease the
pitch>>> less-increase the pitch because you don't want to "level
off")
so speed first, then altitude will come!
Last edited by Hawker-rider : 23rd October 2003 at 12:06.

#10 (permalink)

23rd October 2003, 11:59

sepolnad
Join Date: Oct 2003
Location: Here
Posts: 1

b5, there are different ways to deal with it.


During a situation like an engine failure before reaching V2, but
without any other complication like a secondary system failure,
for example, pilots must follow the recommended SOP for their
airplane. Usually they must rotate at Vr with a continuous motion.
Target pitch attitude have to be established, adjusting initial climb
attitude to maintain a minimum of V2 and positive rate of climb.
IAS can increase (considering landing gear retracted) if the pilot
flies below initial target pitch attitude, but lower than certain
limits a zero climb rate or even an undesirable downward
movement can occur. Performance analyses normally consider a
situation where the airplane will pass the opposite runway
threshold at 35ft and V2, at least. Increasing thrust on remaining
engine(s) is an option, but usually limited to go around thrust.
But now if the situation is beyond a routine emergency drill, its
hard to say the best option to follow. Reduce pitch attitude? Well,
as previously mentioned, the airplane can stop climbing and climb
performance can be severely affected. Increase thrust? Yes, but
not above engine limits unless in case of impending airplane stall
or a possible ground contact. V2 is always a guarantee of safety
(observing some conditions, like bank angle limits, for instance)
and flying below it means a speed close to VMCA or even below
it and its not really a good deal.
Could you assume that the aircraft loading data was
probably incorrect and that the aircraft was actually
heavier than what the figures had been calculated for?
Im sure theres no time to confirm it at least when someone is
fighting to survive during an E/O with a poor airplane
performance.

#11 (permalink)

23rd October 2003, 20:58

backin5
Join Date: Oct 2003
Location: australia
Posts: 12

Thanks to all for extremely thought-provoking discussion!


I'm still not sure that I know what is correct in the situation, but
as most things in this business go, do the right thing on the day
and be hailed a hero, make the other choice and it's all down to
Pilot Error again.
I hope this discussion is not considered over, because if anybody
feels like adding, expanding, contradicting or clarifying further it
will all be very much appreciated!
Thank you!

#12 (permalink)

24th October 2003, 01:15

411A

backin5,

Join Date: Mar 2000


Location: Arizona USA
Posts: 7,508

Large aircraft performance has changed a lot over the years.


With the old 4-engine piston types, runway requirements were
calculated with a 50 foot screen height, and the liftoff speed was
to be no less than V2.
In normal line flying, the aircraft was not rotated until V2 was
acheived. There was no separate Vr, under CAR4b for piston
types.
With most early turbopropeller types, this was also the case.
When jet transports came along, the regulations were re-written
to allow for a 35 foot screen height, and due to aerodynamic
design, a Vr was established, so as to reach no less than V2 at the
lowered screen height.
I can tell you from personal experience flying some of these early
aircraft (especially the non-fan powered B707-321) at heavy
weights, it was common practice to begin rotation at Vr very
slowly, as these aircraft did not perform well in the climb with the
normal rates of rotation used today with more modern types.
Suspect many younger guys today, if they had one of these older
types strapped to their behind, at MTOW, would have eyes the
size of dinner plates at the lack of acheivable climb
performance...and this was with all engines operating.
Very long takeoff runs were required, followed by a surprisingly
shallow climb.
Personally believe that the lowered screen height was selected, so
that these types could be certificated in the first place.

24th October 2003, 10:39

FE Hoppy
Join Date: Sep 1998
Location: Zurich
Age: 41
Posts: 802

25th October 2003, 06:20

LOMCEVAK
Join Date: Sep 2001

#13 (permalink)

Just another thought for your non-normal op.


what is your aircrafts sop for windshear.
probably max power(however its defined by your company),
wings level and fly to the stick shaker. If the world turns to poo
this is probaly your best bet.

#14 (permalink)

V2 is actually the minimum speed at which, with one engine


inoperative, a specified gross gradient can be achieved (3% for 4engined aircraft, 2.7% for 3-engined and 2.4% for 2-engined),

Location: UK
Posts: 252

subject to it being not less than the factors that FE Hoppy states
above.
However, there is one other consideration that must be taken into
account; Vr must be not less than Vmcg. Therefore, if Vr is
determined by Vmcg then V2 may become purely the speed which
is attained at screen height following rotation at the specified
rate; this applies especially at low TOW. This needs to be
remembered in older aircraft where flap retraction speeds are
sometimes based on V2 (747-200 for example). This procedure is
to give a margin above the stall. However, if V2 is determined by
Vr equalling Vmcg then the flap retraction speeds will be greater
than is needed for stall margin.
I know that much of this reply is a little off the original question
but it is important to realise that if you are below V2 then your
climb gradient will be reduced.

25th October 2003, 13:45

backin5
Join Date: Oct 2003
Location: australia
Posts: 12

#15
(permalink)

From my
understanding,
V2 is also a
margin above
Vmca, so if V2
were say,
150kts, then
flying 145kt
means you are
eroding your
protections for
engine failure would probably
need a lot
more rudder,
and some bank
and have to
get the nose
down in a bit
of a hurry.
...and if your
perf fig's were
indeed undercalculated, you
might actually
be a lot closer
to Vmca and
Vs than what
you think...?

26th October 2003, 16:05

Crossunder
Aviator
Join Date: May 2001
Location: Norveg
Posts: 450

#16
(permalink)

There's a
difference
between
minimum V2
(V2min) - and
actual V2
being used for
climbout:
From JAR 25,
Section 1:
[(b) V2MIN, in
terms of
calibrated
airspeed,
may not be
less than
(1) 113 VSR
for
(i) Twoengined and
threeengined
turbo-propeller
powered
aeroplanes;
and
(ii) Turbojet
powered
aeroplanes
without
provisions for
obtaining a
significant
reduction in
the one-engine
inoperative
power-on stall
speed;]
[(2) 108 VSR
for
(i) Turbopropeller
powered
aeroplanes
with more than
three engines;
and

(ii) Turbojet
powered
aeroplanes
with provisions
for obtaining a
significant
reduction in
the oneengineinoperative
power-on stall
speed: and
(3) 110 times
VMC
established
under
JAR 25.149.]
(c) V2, in
terms of
calibrated
airspeed, must
be selected by
the applicant
to provide at
least the
gradient of
climb required
by JAR
25.121(b) but
may not be
less than
[(1) V2MIN;
(2) VR plus the
speed
increment
attained (in
accordance
with JAR
25.111(c)(2))
before
reaching a
height of 35 ft
above the
take-off
surface; and
(3) A speed
that provides
the
manoeuvring
capability
specified in
JAR

15.143(g).]
Autopilot on MEL

I am writing a paper on Cockpit Automation and I would like to know if


some of you could help me.
I would like to know if the event of all autopilots INOP are 'no-go' items on
today's commercial aircraft (737, A320, 767..) MEL (Minimum Equipment
List)? Can the aircraft be dispatched without the AP, even for short-range
flights (non-RVSM flights)?
Thanks a lot in advance.

#2 (permalink)

20th October 2003, 08:50

QAVION

I haven't
looked at our
747-400 MEL's
recently, but 2
out of 3 A/P's
were required
to be
operative. I
can't tell you if
our airline's
744 MEL is the
exception or
the norm,
however.

Posts: n/a

Regards.
Q.

#3 (permalink)

20th October 2003, 09:42

Golden Rivet
Join Date: Feb 2001
Location: The grim north....
Posts: 343

Yes- the aircraft may be dispatched with all autopilots u/s, buts its
highly unlikely/unusual.
According to both 757/767 Boeing DDG's
Except for ER operations, all may be inoperative provided:
c) Enroute operations and approach minimums do not require
their use, and
d) Number of flight segments and segment duration is acceptable
to flight crew.

#4 (permalink)

20th October 2003, 21:19

Max Angle

Also required nowadays for flight in RVSM airspace.

Join Date: May 2001


Location: London,England
Posts: 989

#5 (permalink)

20th October 2003, 22:38

LEM
Join Date: May 2003
Location: The Roman
Empire
Posts: 801

What amazes me is when I see pilots scared of flying with the


autopilot inoperative.

#6 (permalink)

20th October 2003, 22:45

PAXboy
Paxing All Over The World
Join Date: May 2001
Location: Hertfordshire, UK.
Posts: 4,628

Interesting question. In the event of this situation, might we down


the back find that the cruise becomes slightly less smooth? I say
slightly so as not to give offence! But my guess is that the AP
smooths bumps and yaw from wind in a gentle manner. (Now I'll
be in trouble from those who remember cruise as it used to be
when driving Imperial flying boats to Cape Town.)

#7 (permalink)

20th October 2003, 23:46

Jet II

LEM

Join Date: Jan 2002


Location: In the sun
Posts: 845

Quote:

What amazes me is when I see pilots scared of flying with the


autopilot inoperative.
Reminds me of several years ago - got called to the flight deck on
the Queen of the Skies (DC10 for the youngsters) just before
departure and all autopilot systems dead.
As there were two extra flight crew on this sector I suggested fly it
manually back to base - the look of horror on the assembled faces
was something to behold.
Needless to say we cancelled the flight and everyone stayed an
extra night in the Hilton

21st October 2003, 04:12

#8 (permalink)

JW411
Join Date: Oct 2001
Location: UK
Age: 69
Posts: 1,856

Twenty years ago I did London to Los Angeles in a DC-10-30


(single crew) without an autopilot and I don't remember it being
much of a problem. The weather was good at both ends and the
situation kept at least one of us from getting bored.
The lack of an autopilot is not an MEL no-go item on my present
aircraft and I recently did a 4-sector duty without one.
Both events were unusual in this day and age but it is quite nice
to know that I can still fly as well as I did donkey's years ago
before such nice things were invented. Mind you, I tend to handfly below 20,000 ft anyway unless the weather is crap.
"If in doubt, trim to 90 kts and pick a field".

#9 (permalink)

25th October 2003, 09:09

arba

Operation Proc.

Join Date: Jan 2002


Location: beyond PNR .. as
always
Posts: 107

My company MEL/DDG says it is dispatch-able without A/P,


however some OP (operation procedure) there for me to reject to
fly the plane.
It's my butt up there !!

25th October 2003, 10:35

RaTa
Join Date: Sep 2002
Location: Here and there.
Posts: 204

25th October 2003, 17:28

KingoftheRoad
Join Date: Jul 2002
Location: here & there
Posts: 68

#10 (permalink)

Unlike Golden Rivet, our DDG for the B767 requires at least 2 (out
of 3) of the autopilots to be servicable........different airlines
different requirements I suppose.
As far as A320 aircraft are concerned, bus people may correct me
here, I don't think you can dispatch without an autopilot as it/they
are always engaged, even when using the side stick.

#11 (permalink)

There are a lot of misconceptions about the A320 family of


Airbus..... and I'm afraid this is just another one.
The Aircraft is 'hand-flown' regularly by pilots all around the
world, there are two autopilots, and may be engaged at the pilots
will.
The aircraft may be dispatched without any functioning a-pilot.

Roger Miller.

#12 (permalink)

29th October 2003, 13:28

ICT_SLB
Join Date: Apr 2002
Location: The Air Capital
Posts: 474

Don't forget that for many aircraft, the both the Mach Trim and
Yaw Damper reside within the Autopilot (aka Flight Control
Computer or Flight Guidance Computer - two of which are the
norm on most non-CAT III aircraft). Some aircraft can not
dispatch without at least one Yaw Damper operative or, if they can
dispatch, the limiting IAS essentially prevents normal operation.
Similar limiting performance can also be true for the lack of Mach
Trim.

#13 (permalink)

31st October 2003, 01:09

PAXboy
Paxing All Over The World
Join Date: May 2001
Location: Hertfordshire, UK.
Posts: 4,628

Last weekend, I was chatting with the back seater of a Tornado


and a-pilots came up, so I mentioned this thread.
He said that he and his pilot had recently spent a couple of days
with an AWACS crew, to see their side of the picture. They
stooged around for many hours, mostly on a-p, and were able to
observe all the crew at their posts.
When the time came for placing wheels upon the ground, the bod
in the RHS said that he was so tired he was going to use autoland. Apparently, the Tornado pilot was disgusted and ready to
deck him!
(for overseas' readers, it means to punch him so hard that he will
fall to the deck)

#14 (permalink)

31st October 2003, 16:10

Captain Stable

145qrh, please do not repreat your last post or it will be your last
on this forum.

Join Date: May 2002


Location: Who can say?
Posts: 1,656

#15 (permalink)

1st November 2003, 02:02

145qrh

Sorry capt STABLE

Join Date: Dec 2000


Location: The Beach

Edited

Posts: 368

Last edited by Captain Stable : 1st November 2003 at


18:06.

#16 (permalink)

2nd November 2003, 17:02

BlueEagle
Join Date: May 2002
Location: Australia
Posts: 2,252

Taken from the MEL of a well known SE Asian


airline who have operated up to fifty B747-400
aircraft:
For Category III operations all three must be
operative.
For Category II operations one may be inoperative
provided several engineering requirements are
met, mainly concerned with pulling and collaring
the inoperative FCC's FCCServo CBs, OR
To despatch with one or more inoperative (and
that can be all three inoperative) then flight time
must not exceed 3 hours and the aircraft can only
be operated to Cat I limits, also inoperative FCC's
FCC Servo CBs have to be pulled and collared.

Exhaust velocities

First post, Please excuse my ignorance, if I'm asking


something stupid...
I am interested to know any info/website where i can get data
on exhaust velocities behind B737/500 at takeoff. I would like
velocities up to 300m away
What is considered a safe speed for a public access area? eg,
a public road x metres (say) from end of runway experiences
40mph (say) 'jet blast' - is that ok or too much?

Thanks

#2 (permalink)

8th March 2004, 09:42

Paracab

lawnmowerman,

Nice

Tell us a little about the story relating to Joe public


getting too close to the jet blast you are going to

Join Date: Oct 2003


Location: All Over
Posts: 262

report on....

#3 (permalink)

8th March 2004, 11:03

ROB-x38
Join Date: Nov 2003
Location: Australia
Posts: 129

I have a book which suggests at 30m behind a jet aircraft, the


thrust stream will be 25kts at idle and 125kts at full power. I don't
know how accurate that is and it doesn't mention a specific
aircraft/engine.
"Jet blast is powerful"

#4 (permalink)

8th March 2004, 11:18

cloudcover
Join Date: Jan 2004
Location: NZ
Posts: 28

Try it and see

There is a good video of an Air France 747 taking off from


St maarten where people on the beach hundreds of feet
away are blown into the water...sorry no link to the video

#5 (permalink)

8th March 2004, 12:53

av8boy
Join Date: Jul 2002
Location: California USA
Posts: 716

Quote:

Rent the movie "Pushing Tin"...


Knowing suspension of disbelief only goes so far...

#6 (permalink)

8th March 2004, 14:54

DDG
Join Date: Oct 2003
Location: OZ
Posts: 89

8th March 2004, 17:35

Exhaust Hazard Areas

My old training notes for the B737Classic fitted with CFM56b2


powerplants shows the exhaust hazard area extending 100ft(30
meters) from the TAIL of the aircraft at IDLE thrust.At
BREAKAWAY power this extends to 510ft(155meters).At takeoff
power it is 1900ft(579meters).Sorry i don`t have any exact
exhaust veloicities.
Regards DDG

#7 (permalink)

lawnmowerman
Join Date: Mar 2004
Location: cambridge
Posts: 36

Paracab....Ok a bit more info.. Theres one sleepy little midlands


airfield we go to, where theres a public road directly at the end of
the runway, about 200 m back. Not sure what you experts would
say about hanging around in such areas, but it is a public area
and you get a great view and good photos. Now my point is they
are going to start these 737's operations shortly. I would have
thought 200m in the line of fire was pretty close for a public area?
DDG - sorry for being ignorant but whats 'breakaway' power? At
the end of the runway at start of takeoff does the 'takeoff' power
apply?

#8 (permalink)

8th March 2004, 18:41

LEM
Join Date: May 2003
Location: The Roman
Empire
Posts: 801

Can't help with the speeds for the moment (I'm in a hurry) but
the mass of air exausted from a CFM56-3-B1 is 297 Kilograms per
second (note: just one engine)...

#9 (permalink)

9th March 2004, 06:45

DDG
Join Date: Oct 2003
Location: OZ
Posts: 89

lawnmowerman,
Breakaway power is the amount of power used to start to get an
aircraft to taxi,(ie breakaway from standstill).
This is a Boeing term used within their manuals.
As each environment/handling pilot will be different this figure i
imagine would be an average used as a guideline.
The take off power hazard area i listed is for static take off power
as used for ground running purposes,so in your example it would
apply from the end of the runway(panio keys) .
If you have access to the aircraft mantenance manual the figures
and warnings are listed in mm71-00-00-201 Engine ground safety
precautions.Note the AMM doesn`t list exhaust velocities within
the hazard area.
Regards DDG

#10 (permalink)

9th March 2004, 14:39

OverRun
Prof. Airport Engineer
Join Date: Oct 2000
Location: Australia (mostly)
Posts: 489

Lawnmowerman
Go to http://www.geocities.com/profemery/aviation.html
Down the page - in the ACN chart section - is a chart of the 737800 jet blast at TAKEOFF THRUST, not breakaway thrust.
Typical rules are:

maximum wind velocities which people, objects and buildings in


the vicinity of an aeroplane may be subjected to must not be
more than:
(a) passengers and main public areas, where passengers have to
walk and people are expected to congregate . 60kph;
(b) minor public areas, where people are not expected to
congregate . 80 kph;
(c) public roads . 50kph where the vehicular speed may be 80 kph
or more, and . 60kph where the vehicular speed is expected to be
below 80 kph.
(d) personnel working near an aeroplane . 80kph;
(e) apron equipment . generally not in excess of 80kph;
I got to tell you even 50 kph feels like a lot more. Saw some
tests recently using a hand-held anemometer from Bureau of
Meteorology. Here is what was written: "Pilot was requested to
use maximum power takeoff consistent with safety for the
purpose of measurement. 737-300 aircraft. Maximum measured
jet blast was 45 kph with instantaneous gust to 50 kph. Note that
there was significant dust/sand blast, and without the benefit of
the anemometer it would have felt to be a higher blast velocity
than actually measured."
In normal speak it really feels like you're being blasted even at
these modest blast speeds. This test wasn't unsafe at all just
[forgive me for this] a real blast.
Boeing jet blast data for the 737-300 (and 400 series) (D658325-2; July 1990) shows 56 kph blast contour to go out to
155m for breakaway thrust. The 56 kph takeoff power contour
goes to 579m (geez - same as what DDG was quoting - nice to
find two technical documents agreeing). Back to the tests "The
breakaway thrust contour is consistent with the measured
maximum blast velocity, and supports the use of breakaway
thrust for blast design".
So 200m would seem to be pretty reasonable. There are airports
which have got public areas which are closer. Some of them stick
up blast walls, some don't. Some do, and the photographers got
annoyed because they can't see the aircraft. Some don't and noone complains about the blast.
Just looked at the movie that ROB-x38 linked to. Great clip.
The van looks to be about 20m behind the tail - maybe a little
less. From the 737-800 blast graph, and assuming that the United
had engines wound up to takeoff thrust for the demonstration,
that works out to be 300-325 km/hr jet blast. Up close like that,
jet bast is a killer.
That blast speed is faster than then the winds in a bad cyclone.

Explains why the van got trashed. Wonder if the van was a rental.

#11 (permalink)

9th March 2004, 17:46

lawnmowerman
Join Date: Mar 2004
Location: cambridge
Posts: 36

Thank you very much for your detailed replies. Thats just what I
was after.
Looks like I might need some goggles, and stand to one side in
the future!

#12 (permalink)

10th March 2004, 00:28

Mark 1
Join Date: Aug 2000
Location: L.A. CA USA for
the time being
Posts: 665

At the nozzle exit, the velocities will be around 1400-1500 ft/sec


at the hot nozzle and around 1000 ft/sec at take-off power (just
below locally sonic conditions).
For the long cowl mixer nozzle type engines the hot jet rapidly
diffuses with the bypass flow.
Further downstream the exhaust will diverge at about 7 degrees.
I don't know if this would hold up for 300m, but if it did the jet
would be over 30m in diameter, so about 400 times the area at
source and so 1/400th the velocity.
I recall a test some time ago for designing an engine test bed
where a pitot rig was used to measure the blast at about 50m
behind a jet engine at full power. All of a sudden all the sensors
read zero due to the entire rig complete with several blocks of
concrete being blown into the next field.
Hope this helps.

#13 (permalink)

14th March 2004, 07:45

mstram
Join Date: Mar 2001
Location: Toronto, Ont,
Canada
Posts: 211

Mark 1
>Further downstream the exhaust will diverge at about 7
degrees.
I don't know if this would hold up for 300m, but if it did the jet
would be over 30m in diameter, so about 400 times the area at
source and so 1/400th the velocity.
If, hypothetically the exhaust did *NOT* diverge, what would the
calculation be to determine the distance where the velocity drops
to zero ? Just approximately would be interesting enough for me.
I'm guessing that the air density / viscosity would be part of the
calculation and probably the friction over the ground. What if we
ignore the friction over the ground ? .. is it just a question of the
velociity, viscosity, density all part of some kind of formula ?
Mike

#14 (permalink)

15th March 2004, 14:19

Mark 1
Join Date: Aug 2000
Location: L.A. CA USA for
the time being
Posts: 665

If the jet did not diverge then it would go on forever.


The equation for mass flow in a stream tube is:
Density * Area * Velocity = Constant
So not a good assumption for calculations.
My guess would be 100-200 nozzle diameters to dissipate the jet
from a critical nozzle.

#15 (permalink)

15th March 2004, 15:32

Ranger One

cloudcover,

Join Date: Nov 1999


Location: Mk. 1 desk at present...
Posts: 263

Try:
http://www.storeitonline.nl/funny/re...depart-09.mpg
R1

16th March 2004, 07:24

#16 (permalink)

cloudcover

Ahhh thats the bugg*r, cheers Ranger

Join Date: Jan 2004


Location: NZ
Posts: 28

If that doesnt put you off nothing will!

Approach Catergory

There is some dicussion at my company concerning


which Catergory our Dash-8s fit into on a circling
approach.
Our company profile is to fly the aircraft clean
(gear and flap up) at 140kts until visual with the
runway. When and if visual the flap and gear are
extended and the speed reduced to 120 kts.
Normally we consider the Dash 8 to be a Cat B
aircraft but on the Jeppesen plates there is a note
saying 135kts max under circle to land minima Cat
B. Does this mean that we are Cat C because of
our company profile?
My belief is that the aircraft is still considered Cat B

since the 135 kts max applies to the visual


manoevering segment of the approach.
Thanks

#2 (permalink)

26th March 2004, 03:09

None

Use the Jeppesen speed/MDA chart

Join Date: Jul 2000


Location: West
Posts: 364

I think your question is must you use


the higher minimum if the higher
airspeed is used only before
maneuvering.
When you begin an approach and plan
to circle, I believe that you use circling
minima. If you are at 140 kts, yet fly to
the MDA for 120 kts, that seems to be
contrary to the Jepp published minima.
In fact, this would be contrary to what
many operators are doing these days.
That is, many have the pilot fly to the
highest minima published for the
circling approach. The reason for this is
because otherwise a circling approach
delivers the aircraft to low speed
maneuvering at low altitude, in poor
visibility at a time of high cockpit
workload. At this point your company
wants you to configure and change
airspeed while maintaining visual with
the runway?

#3 (permalink)

26th March 2004, 20:20

OverRun

Aircraft approach category

Prof. Airport Engineer


Join Date: Oct 2000
Location: Australia (mostly)
Posts: 489

I'm not sure if I've got the right sort of category here. Are we
talking about 'aircraft approach category' which is part of the FAA
definition to classify the airport? If not, and it is some instrument
flight category, then please disregard the rest of this post.
If it is the FAA airport classification, then the Dash 8-300 is
classified by the FAA (reference: AC150/5300-13 Airport Design)
as A-III, which is aircraft approach category A.
The basis for their classification is the aircraft approach speed,
and the limit for aircraft approach category A is <91 knots. The

aircraft approach speed is defined as 1.3 times the stall speed in


the landing configuration of aircraft at the max certified landing
weight. I'm not sure what the stall speed of the Dash-8 is, but the
FAA has the approach speed stated in their database as 90 knots.
The Cat B limits are 91-<121 knots and Cat C is 121-<141 knots.
Even if the aircraft approach category was to change from A to B
for some reason (while staying aircraft design group III), there
would be no change in the airport standards (FAA Table 1-1 in
AC150/5300-13). It is only moving from Cat B to Cat C (while
staying aircraft design group III) that means a step up in terms of
the runway strip (the FAA call this the runway safety area). The
runway now has to be less undulating and have flatter gradients.
However at 9 out of 10 airports, youd be hard put to pick the
difference. The 10th airport is probably one of those spectacular
Dash-8 airports up the side of a mountain or similar.

27th March 2004, 13:42

#4 (permalink)

alf5071h

I believe the following information to be reliable in that it


originates from ICAO PANS-OPS.
Join Date: Jul 2003
The design of takeoff, approach, and landing procedures is
Location: An Island Province governed by the aircraft classification (A - E). This classification is
Posts: 697
based primarily on the threshold speed (Vat) which is 1.3 x stall
speed (Vs) for the maximum certificated landing mass. Note that
some operators have lower landing wt certification for noise / enroute / landing fee reduction. I dont know of anyone who has
done this for approach criteria, but there has been discussion
between an operator and authority re landings on short / low
strength runways. Thus same version of aircraft as well as
different versions may not have the same approach category.
Also note that some aircraft will have a higher category than
specified by Vat due to the speeds that have to be flown during a
procedure because of the aircraft design (N.B. abnormal
configuration operations / icing limits).
Thus for a MLW Vat of 120 kts the aircraft would be Cat B, but all
categories also considers the maximum takeoff speed (165), the
range of speeds for the initial approach (120/180, but 140 for
reversal / racetrack), the range of speeds for final approach
(85/130), maximum circling speed (135), and the maximum
speeds during intermediate and final missed approach (130 and
150). All examples ( ) relate to Cat B.
Some approaches are specifically defined for lower speeds, these
must be followed. From Captain Smileys question and his
reference to a speed limit in Jeps I suspect that this is the limiting
factor. Therefore whilst the aircraft is Cat B, it would be incorrect
to fly the 140 kt SOP speed on any procedure with a lower speed
restriction. Note that the procedure is owned by the national

authority of the country who licenses the airfield, thus it may be


inappropriate to seek a dispensation from your own authority or
inspector as they may not have the relevant information
Also beware that for procedures designed by TERPS (US) whilst
generally giving the same obstacle clearance, the circling and
maneuvering areas may differ significantly; e.g. 767 CFIT at
Busan Korea during circling procedure.
Edit: threshold (Vat) replaced touchdown
Last edited by alf5071h : 3rd April 2004 at 20:20.

27th March 2004, 15:33

redsnail
PPRuNe Handmaiden
Join Date: Feb 1997
Location: Duit On Mon Dei
Posts: 3,374

30th March 2004, 07:54

#5 (permalink)

This is testing the ol' memory a bit. I flew the Dash 8 briefly in
Oz. It's a Cat B aircraft. If we wanted to fly it faster for whatever
reason in the approach/circle to land then we'd have to use the
Cat C requirements so we remained in the protected area.
Have a squizz at the charts to see what the circling minima is and
stay within that for Cat C. That should keep you safe and away
from the ground/terrain.

#6 (permalink)

OzExpat
PPRuNeaholic

Join Date: Jun 2000


Location: Port Morbid
Posts: 3,257

When the Pans Ops speed classification system was introduced,


both here and in Oz, there was a fair degree of confusion on this
point. The basic situation here is that, if you can't meet ALL the
requirements of the nominal category, you must use a higher
category that allows you to meet all the speeds. Circling was
certainly the major issue for many aircraft that are nominally in
Cat. B, so they all had to use Cat. C.
And, before anyone asks, no you can't use a lower category on
the basis of speeds for actual configurations. I understand that
ICAO is about to change the basis of the speed categories in the
near future, but it isn't likely to change the speed requirements in
any of the segments.

31st March 2004, 13:16

Crossunder
Aviator
Join Date: May 2001

#7 (permalink)

For a circling approach the DHC8 can be flown using CAT A speeds
(we use circling speed of max 109 knots for precision circling, and
the minimums are determined "-100 minimums" because they are
aircraft specific). It really doesn't matter which aircraft you fly; it's
the speeds you use that matter. On a CAT B approach you must

Location: Norveg
Posts: 450

stay below the maximum of 135KIAS when reaching minimums.


Note that the Categories with reference to 1.3 Vs are different
from the categories concerning the approach itself. If an
apporoach is labelled 'CAT B only'; any aircraft can fly it, provided
they'll remain at or below 135KIAS (or, say, 130KIAS for the
MAPt).

#8 (permalink)

1st April 2004, 02:44

Bomber Harris
Join Date: Dec 1999
Posts: 178

crossunder. what you say makes a lot of sense. but i would tend
to lean towards what 'alf' posted. when this is analysed at the
'subsequent board of enquirey' the legal definitions will be what
counts. i would take adhereing to those as the first and formost
rule. i.e. if the 1.3 rule makes an a/c cat b but it can be safely
operated at cat a speeds....then it is still cat b because thats what
the definition says. however, what redsnail says rings some loud
bells for me too. i think it says somewhere that if a condition exist
which will not allow you to keep to speeds...in my example...cat b
then the approach must be done to cat c minima.
apologies if i have misinterpreted what you were saying and
answered the wrong question!!....complicated issue

#9 (permalink)

1st April 2004, 08:15

OzExpat
PPRuNeaholic

Join Date: Jun 2000


Location: Port Morbid
Posts: 3,257

Quote:

If an apporoach is labelled 'CAT B only'; any aircraft can fly it,


provided they'll remain at or below 135KIAS (or, say, 130KIAS
for the MAPt).
Definitely not true and potentially quite a dangerous idea to try to
circulate. It demonstrates a complete ignorance of Pans Ops Volume
1, if nothing else. It doesn't matter what speeds any particular
aircraft can achieve, if below any of the speeds for the nominal
category.
I know that the Dash-8, for example, is a very flexible aeroplane.
No doubt about that.
But lets take the example of a very flexible type of aircraft that is
nominally in Cat C. It doesn't matter if it can maintain 130 KIAS or
below, from the FAF to the MAPt. It also doesn't matter if it can
maintain 130 KIAS or below for circling. This is because the
determining factor for this aeroplane is the basic 1.3 Vs at max
certificated weight.

The same sort of consideration applies to all other aircraft in all


nominal categories. Thus, getting back to the Dash-8, it is a Cat B
aeroplane, regardless of what aspects of Cat A it can achieve.
#10 (permalink)

1st April 2004, 08:46

126.9

Category B

Join Date: Jun 2001


Location: Europe
Posts: 609

In accordance with the All Weather Operations (JAR-OPS Subpart


E)
Your aircraft is Category B, come hell or high water! The Category
B classification is based upon the Vat (ie: across threshold and
NOT at touchdown as suggested above) being 1.3 times the
stalling speed (Vso) or 1.23 times Vs1g at MCLM in the landing
configuration. You cannot change that.
With regards to which minimums are applicable: your company
needs to operate according to the manufacturers certification
speeds and should make use of the procedures and techniques
certified by the manufacturer for circling (and other) approaches
in order to remain within the bounds of Category B operations.
Flying a higher speed however, makes the higher circling minima
applicable.

#11 (permalink)

1st April 2004, 09:34

Oktas8

Operating to higher category

Join Date: Jun 2001


Location: NZ
Posts: 387

A question - and sorry for re-starting a tired thread; I've just


come across it.
The rules in this country (NZ) allow an aircraft to fly to a higher
category, provided ATC knows about it. But how would one go
about telling them?
"Request ILS 23L, and aaah we'll be Cat C today."
AFAIK one simply operates to the category one feels like operating
to (higher not lower), and obeys speeds & minima for the new
category regardless of aircraft type - and who cares?

#12 (permalink)

1st April 2004, 13:39

126.9

Oktas8

Join Date: Jun 2001


Location: Europe
Posts: 609

The rules in that country do nothing of the sort! Cat B is Cat B


and Cat C is Cat C etc. and you are not able to re-classify the
aircraft just because you feel like it! Read the rules again!
Of course there is nothing stopping you flying whatever speeds
you like, as long as you apply the correct minima. And why would
you need to tell ATC about that?

#13 (permalink)

2nd April 2004, 12:58

OzExpat
PPRuNeaholic

Join Date: Jun 2000


Location: Port Morbid
Posts: 3,257

Most countries that I'm aware of require you to tell ATC. It is


relevant to their assessment of the cloud base in clearing you for
the approach. Pans Ops isn't the only yardstick in operations!
The post by 126.9 is interesting from the point of view of the
design of the procedure. If what was posted is correct, there is no
flexibility for a higher category, based on the maximum speeds
within a specific category.
For example, I once had a bit of a dilemna with the F28 because,
in theory, it could achieve Cat B in everything except circling. That
made it Cat C. That's the category that's used by this type to this
very day but, if I understand the post by 126.9 correctly, there is
one ICAO-contracting State that does not abide by this. I just
hope that the AIP (or Regulations) reflects that.

#14 (permalink)

3rd April 2004, 09:30

Oktas8

Thanks for the replies.

Join Date: Jun 2001


Location: NZ
Posts: 387

126.9 - hmm, fairly severe misunderstanding there.


No-one's talking about reclassifying an aircraft, but I did say
"flying to a higher category", meaning "flying whatever speeds
you like, as long as you apply the correct minima". Hope you
recognise the similarity between those two quotes, mine & yours!
Unfortunately, still confused. Your comment re ATC makes sense
OzExpat - have you ever actually told ATC you're operating to a
higher category speed / minima?
O8

3rd April 2004, 20:19

#15
(permalink)

alf5071h
Join Date: Jul 2003
Location: An Island Province
Posts: 697

Not having
flown the
flexible
option
described
above, I
assume that
the
notification
to ATC also
allows for the
appropriate
straight in
maneuvering
allowance
higher MDA
and greater
visibility to
counteract
the higher
speed.
However this
may not
apply to
circling.
Previously I
referred to a
CFIT accident
in Busan
Korea, a
resume is
here:- Busan
3.pdf.
Although the
error in this
accident was
the
assumption
that PANSOPS circling
areas applied
(4.2nm rad
from t/h. Cat
C) whereas
the airfield
was actually
charted to
TERPS (1.7
nm rad from
t/h. Cat C),

the issue of
flight outside
a safe area
may also
arise with
increased
airspeed.
A turn at 20
AOB 180 kts
(Cat C) gives
a radius of
approx 1.4
nm; thus
with TERPS
this only
gives a small
safety factor
for error,
wind, etc;
lower bank
angles
reduce this
margin.
Flying at Cat
D speed
205 kts the
rad of turn is
1.8 nm at 20
AOB, which
exceeds the
TERPS area.
These
changes
apply to
PANS-OPS in
a similar
way, but the
safety area is
larger. Thus
the aircraft
category
speed limit
must be
respected
and there is
also a
minimum
AOB to
consider.
Many of the

issues in this
thread are
presented in
an excellent
booklet
From
Takeoff to
Landing by
Olle Akerlind
who worked
for SAS
performance
dept; there
was a limited
reprint by
Honeywell
last year, but
it is not on
the web.

#16
(permalink)

4th April 2004, 05:43

OzExpat
PPRuNeaholic

Join Date: Jun 2000


Location: Port Morbid
Posts: 3,257

Oktas8...
no, I've
never done
that and I
suspect that
nobody else
has done it
either - at
least, not in
the course of
normal
operations. I
suspect that
the main
reason for
this provision
is
consideration
of an
abnormal
operation. An
example of
this might be
where a
particular

aircraft
cannot use
all the flap
normally
required for
landing thus, a
higher
approach
speed is
likely to be
necessary.
With advance
notice of
this, ATC can
assess
weather
conditions
against the
higher MDA
and advise
the pilot
accordingly.
If the
weather isn't
good enough
for that
higher MDA,
the pilot has
a bit more
time to plan
for an
alternate
approach, or
alternate
aerodrome.
alf5071h... I
was going to
download
that Busan
report, but
got a
message
saying I need
to download
some sort of
graphic
rendering
utility.
Couldn't be

bothered
with that on
this low
speed, high
cost dialup
access.
Anyway, I
have an idea
that the FAA
is starting to
rethink the
size of TERPS
circling
areas, so we
may see a
significant
enlargement
at some
stage, so
that they'll
be more in
line with
Pans Ops
areas.
B737 go around sequence

If I take off with flaps 15 and have an eng failure I'll wait till the established
altitude to retract flaps on schedule.
Now, let's say I go around, set flaps 15 and THEN have an eng failure: shall I
retract immediately the flaps to 1 or shall I wait till the acceleration altitude?

11th May 2003, 04:33

PifPaf
Join Date: Feb 2003
Location: Brasil
Posts: 39

#2
(permalink)

In my
opinion, you
must
maintain the
flaps until
the level off
altitude.
After the go
around
procedure
was initiated,
we could
consider as

we were
taking off.
So, if you
chose to
takeoff with
flaps 15, you
would wait.
In the same
way, if you
already
began the go
around, you
must procced
with the
selected
flaps.
Any contrary
opinions?
Regards,
PP

#3 (permalink)

12th May 2003, 01:11

CI54

Hello,

Join Date: Feb 2003


Location: Kuala Lumpur
Posts: 34

I agree with you Piff Paff. In fact the Boeing FCTM specifically
mentions it as such; In case of engine failure during go-around,
treat it as a flaps 15 takeoff.
The reasons could be numerous but one which I can think of is
the speed. The Vref for 2 engine approach is either Vref flaps 30
or 40 (this is B737) + 5. If one is on single engine approach, the
Vref is Vref 15 + 5. The difference in speed is quite significant. In
a single engine go around, the higher speed is used to
compensate the loss of lift through the flap retraction. On a two
engine go-around, if one loses an engine, it may be that he has
not reached the 160kts ias the flight director is asking him to
maintain with flaps 15. So, if flaps is further retracted to flaps 1,
the resultant decrease in lift will be, I believe, significant. You may
not be able to even achieve a positive climb gradient, let alone
the required climb gradient.
I hope this helps. As usual, I stand corrected...

12th May 2003, 07:45

Bokomoko

#4 (permalink)

...In case of poor climb perfomance increase speed to Vref + 15


and retract flaps to 1. That speed is approximately equal to V2 for

Join Date: Nov 2000


Location: sunny country
Posts: 101

flaps 1 and must be used in case of engine failure prior to go


around.

#5 (permalink)

12th May 2003, 19:50

LEM
Join Date: May 2003
Location: The Roman
Empire
Posts: 801

That's the information I had, Bokomoko!


But After a few years I can't find the source for that information:
In case of eng failure in final, if you decide to go around, retract
the flaps to 1 !!!
Can you tell the source of that information?
LEM

#6 (permalink)

12th May 2003, 20:27

timzsta
Join Date: Feb 2002
Location: UK
Age: 32
Posts: 592

If I remember back to my ATPL PoF and Perf exam in Feb this


year, is it not the case that the more flap you have out the lower
the climb gradient will be. So by going to Flap 1 we will increase
the climb gradient and hence better obstacle clearance and meet
the required climb gradient on go-around (seem to remember its
about 2.4%). Stand corrected, as I am still only a student!!

#7 (permalink)

12th May 2003, 20:43

Aviation Trainer too

edited for error in judgement: flaps 1 is the one!!

Join Date: Oct 2002


Location: where ever I lay
my head
Posts: 142

12th May 2003, 22:21

Bokomoko
Join Date: Nov 2000
Location: sunny country
Posts: 101

#8 (permalink)

LEM,
Information describing procedures in case of "Engine Failure In
The Landing Configuration" was in the old 737 Operations Manual,
Volume 1, Non-Normal Procedures section (expanded
information), page 03.05.07. Anyway, there's no mention about
Engine Failure after starting a G/A, but a maneuver in the event
of a G/A with an E/O before the decision point (in my point of
view even more restricted)
When the new 737 manuals edition arrived, all information
concerning that maneuver is in the FCTM, but without an
explanation about Vref + 15 and V2 for F1...Well, my manual can
be outdated, since I've not been flying 737 lately.
Regards,
Bkmk

13th May 2003, 01:54

pancho
Join Date: Feb 2000
Location: Ireland
Posts: 45

13th May 2003, 05:38

LEM
Join Date: May 2003
Location: The Roman
Empire
Posts: 801

13th May 2003, 08:40

Pickle
Join Date: May 2003
Location: offshore
Posts: 6

#9 (permalink)

Engine failure procedure in the G/A is as mentioned, deal with it


as a F15 T/O.
Engine failure on approach can be a subtle and dangerous event.
However, there are essentially two ways to deal with it. In IMC it
is a mandatory G/A. At normal landing weights Vref F30/40 is
more or less equal to V2 F15, therefore treat it as a F15 T/O for
accel. In VMC some companies (and Boeing) allow the approach
to be continued. In this event you must set G/A power, select F15
and accelerate to at least Vref +15 (sometimes known as the
"High White Bug") +5. If at a later stage on the approach you
decide to G/A, now the call will be for F1 as in the 1 ENG INOP
approach and accelerate as per sched. Hope it never happens!

#10 (permalink)

Thanks, Bokomoko! I found it!


So, let's sum up:
In case of ENGINE FAILURE IN THE LANDING CONFIGURATION
Boeing says: " ...In the event of a go around, maintain VREF+15,
retract flaps to position 1 (!) ..."
So that's what I'll do. We know it takes a lot of time for the flaps
to move from 30 to one, so no danger of stalling while
accelerating while pulling up!
Thanks everybody!
LEM

#11 (permalink)

Yes, but the first priority is to select Flaps 15 and increase the
thrust and get to Vref 30/40 + 15 (which = Vref 15).
Then, unless you are stabilized (above approx 1000' RA) or very
close to the runway (below approx 100' RA), it is advisable to GA.
Maintain the Vref 30/40 + 15 speed and select Flaps 1 for GA.
(On some models the additional speed is ref +20 knots).
If an engine failure occurs during a GA, use normal GA procedures
(select Flaps 15) and verify max GA N1 is set.

13th May 2003, 22:10

#12 (permalink)

LEM
Join Date: May 2003
Location: The Roman
Empire
Posts: 801

I like your idea, Pickle, to split the sequence in two: first, increase
to vref+15 and flaps 15 on the glide, and THEN pull up retracting
the flaps to 1!

#13 (permalink)

14th May 2003, 15:47

Pickle

Yes LEM,

Join Date: May 2003


Location: offshore
Posts: 6

I think it is OK to initiate the GA as soon as required (selecting


Flaps to 15), especially if you are getting low on altitude. I think
with max GA N1 selected you will find the speed still increasing
while you slowly pitch up. We all know the aircraft will climb with
Flaps 15 on one engine no trouble at all.
What I meant is not to select Flaps 1 until you have the required
speed. I don't recommend selecting Flaps 1 straight from Flaps
30/40. Something may happen in between or distraction, and
then you find you are below your safe speed margin.

#14 (permalink)

16th May 2003, 17:51

john_tullamarine
Moderator
Join Date: Apr 2001
Location: various places .....
Posts: 3,767

29th November 2003, 06:02

Popolama
Join Date: Nov 2003
Location: NXX 50.5 E010 13.1
Posts: 30

A couple of considerations (a) standardisation is extremely important for such high workload
but non-routine operations
(b) the underlying consideration is matching speed to
configuration. Easiest way to check out this aspect is to look at
the QRH and compare V2 and Vref data. The standard procedures
then fall into place quite easily.

#15
(permalink)
go around

now let s say


you made a
go around
and you had
an engine
failure after
the go around
while
climbing.what

procedure
should you
follow? the
normal
missed
approach
procedure on
the approach
chart or
should you go
for the one
engine inop
compagnie
procedure
(exp:Rwy hdg
ctc atc) ?

2nd December 2003, 23:41

Thinair
Join Date: Oct 2003
Location: Belgium
Posts: 4

#16
(permalink)

Vref F40 +
15kt ~ VMan
F15 ~ V2 F1
Situation:
bugs set for
F40, go
around,
retract flaps
to F15 Then
engine
failure...
So If you:
-need climb
performance:
New target
speed=outer
bug (Vref F40
+ 15kts) ,
then retract
flaps to F1.
You are now
flying V2, so
your max
bank angle is
limited to
15deg.(plus
'overbank')
The turn
radius

however will
be large due
to the shallow
bank angle.
(As during an
engine failure
on take-off)
-need turn
radius : New
target speed
is (again)
outer bug,
keep flaps 15.
Your are
flying Vman
F15, so you
can use
normal bank,
which gives
you a good
turn radius,
but reduced
climb
performance
due to the
drag being
greater for
F15 than for
F1, AND the
greater bank.
So for a
close-by
obstacle that
you cannot
avoid by turn
(e.g. a low
hill range), I
would choose
climb
performance
(F1).
And for a high
obstacle that
can be
avoided by
turn, a better
turn radius
seems
appropriate
(F15)

Hope this
helps (Hope I
didn't mix up
the numbers)
tail stall recovery

Does anyone know why the recovery technique of a stalled horizontal stab
(icing conditions) calls for pulling on the yoke?
It would seem that pulling on the yoke would only increase the AoA of the
stalled horiz. stab...
Looked everywhere and I can't find a detailed answer.

27th November 2004, 23:18

PickyPerkins
Join Date: Jul 2002
Location: 40N, 80W
Posts: 169

#2
(permalink)

I have never
flown with a
stalled
horizontal
stabilizer, and
it sounds to
me like a
maybe nonrecoverable
situation with
uncontrolable
increasing
pitch down.
But here is
my guess.
For a plane
flying straight
and level the
lift on the
wings is up
while the lift
on the
horizontal
stabilizer is
down. This is
required for
pitch stability,
and is a
consequence
of the center

of lift on the
wings moving
forward as
the angle of
attack
increases.
Without the
downward
force on the
tail the
aircraft would
be unstable in
pitch (unless
it had a
canard).
So if the tail
stalls due to
icing the
downward
"lift" needs to
be increased
somehow,
and I guess
up elevator
might be one
way of doing
this. Pulling
on yoke
should
decease the
AoA on the
horizontal
stabilizer
Maybe
someone who
really knows
the suject can
expand on
this.
Cheers,

27th November 2004,


23:33

batty

#3 (permalink)

In straight and level flight the lift on the horizontal stabilizer isnt
always down , it depends on where the CofG is. In order to

Join Date: Feb 2001


Location: South East
Posts: 202

increase range an aft CofG is preferable in the cruise so that there


is lift on both the wing and the stabilizer. If the stabilizer is
providing a downward force it in effect acts as an inrease in
aircraft weight and thus a decreased range. An aft CofG does
decrease the longitudinal stability though.

27th November 2004,


23:36

Mad (Flt) Scientist


Join Date: Sep 2002
Location: La Belle Province
Posts: 1,519

#4 (permalink)

You need to somehow generate the required amount of lift on the


tail surface to trim out the tail-off pitching moment. If the ice on
the tail leading edge is causing the tail to stall, you might - and
only might - be able to increase the elevator angle (by pulling
back on the stick/yoke) and also decreasing the actual stab angle
- that way you might manage to give a more aft-loaded tailplane
lift distribution and so unstall the tail.
The problem I can see is that the process of unstalling a surface is
not the reverse of stalling it - usually there's some hysteresis - so
you'll likely have to reduce the AoA at the tail to somewhat lower
to unstall the tail than the angle it stalled at.
That would seem also to be quite a slow process - trying to find a
new STAB/elevator combination isn't immediate, and if the tail is
meanwhile stalled other, bad, stuff will be happening.
If pulling back on the yoke does something - and I dont see how
it could be guaranteed - you might be able to start a niose-up
pitching motion. The dynamics of the plane's rotation would tend
to reduce the angle-of-attack on the tail, which might help unstall
the tail, giving long enough to input a nose-down stab input.
It all sounds very very dicey though.

28th November 2004,


02:50

oldebloke
Join Date: Apr 2004
Location: vancouver
oldebloke
Posts: 261

28th November 2004,


03:37

Milt

#5 (permalink)

Interesting concept....The first thing to call/select ,in the event of


tailplane stalling,is to reduce flap setting(back to where they
were)as the selection of the flaps rediected the airflow into the
stall angle...Viscount/DC4 cases...
As has been said before the increase in backstick can only
increase the curvature of the down lift stabalizer..

#6 (permalink)

Horizontal tail stalling is a very rare event. Perhaps we don't hear

Join Date: Oct 2003


Location: Canberra Australia
Posts: 1,186

about it because it is most likely to have terminating effects.


Ice accretion on the wings will most often occur and be a problem
before enough builds on the tail. When it does build up on the tail
the ice will be depleting the down lifting characteristics of the tail
surfaces rather than causing them to stall in the usual sense. So
with ice build up on the leading edges you should still have some
control with the trailing elevator/s and also even if it is a full slab
tail.
Incidently FBW and artificial stability will be a way we can design
for a zero tail load or even an up load on the tail for cruising
flight..It's happening with military aircraft and close for nonmilitary using artificial stability. But the elevators will then have a
very busy time preventing runaway in pitch.
Can anyone describe an actual case of horizontal tail stall or
major loss of tail downforce resulting from ice build up before it
becomes a major problem with the wings.?
Maybe on an aircraft only fitted with wing de-icers.

28th November 2004,


10:48

#7 (permalink)

alf5071h

bobrun ice in the Cayman Islands? But seriously we must not


forget ice at higher altitudes and in all parts of the world.
Join Date: Jul 2003
The main difference in recovering from a tailplane stall in icing
Location: An Island Province conditions is that generally these stalls are negative stalls, thus
Posts: 697
the control inputs are reversed.
My reference for the following is Think Ice 2000 from BAE
SYSTEMS.
A negative tailplane stall is more associated on aircraft with
mechanical elevator controls.
Normally, the tailplane creates a force (lift) in the downward
direction to balance wing and fuselage pitching moments. Under
normal conditions, without ice accreted, aerodynamic pressures
above and below the elevator are roughly equal and thus create
no significant control surface hinge moment.
Extending the flaps increases the airflow downwash angle from
the wing and increases the tailplane AOA more negative. For a
given flap setting, the AOA on the tailplane becomes more
negative with increasing speed because of the reduced AOA of the
wing (more nose down, more tail up).
Therefore, at higher flap angles and high airspeeds the wing stall
margin is increased, but the tailplane stall margin is reduced.
The occurrence of stall on any aerofoil contaminated by ice almost
always occurs at a lower angle of attack than a clean aerofoil;
hence any ice accretion on the tailplane reduces the tailplane stall
margin further. It is worth emphasizing that ice can form on the

tailplane at a greater rate than the wing, primarily due to its


relative small size and smaller leading edge radius. This can lead
to a significant build up of ice which is not evident from
observation of ice accretion on other areas of the airframe.
From the pilots point of view, the most important characteristic of
a tailplane stall is usually the suddenness and magnitude of the
nose down pitch change most often accompanied by unusual stick
forces, with the control column moving towards the forward limit.
Link to a larger extract and diagram from Think Ice 2000 : see
Tail Ice / Tailplane Stall.
The FAA has a good training aid / video; I dont have a link, but it
was distributed on a DVD.
Edit: Delete reference to FAA DVD; substitute NASA Multimedia
CD-ROM "A Pilot's Guide to In-Flight Icing" NASA Glenn Research
Center, Icing Branch, 21000 Brookpark Rd. MS 11-2, Cleveland,
OH 44135, http://icebox.grc.nasa.gov
Last edited by alf5071h : 28th November 2004 at 12:07.

28th November 2004,


17:44

411A
Join Date: Mar 2000
Location: Arizona USA
Posts: 7,508

#8 (permalink)

Yes, Milt, I believe I do recall an incident of tailplane icing, which


resulted in the loss of the aeroplane.
IIRC, it was a Swedish(?) registered Vickers Viscount, on
approach...some time ago.
Apparently the Viscount suffered from this problem under certain
conditions, and the new operator was unaware of the previous
difficulties.
I do personally know that the DC-4 had similar difficulties, altho
to my knowledge, no aircraft was lost due to this.
When the DC-6 was designed, the same wing (with very minor
increased washout toward the tip) and horizontal tail surfaces
were used, but the DC-6 had combustion heaters for anti-icing,
which were very effective.

28th November 2004,


19:20

John Farley
Do a Hover - it avoids G
Join Date: Oct 1999
Location: Chichester West
Sussex UK

#9 (permalink)

In the early days the Hawk could suffer from tailplane stalling
gear and flap down given a tad of ice (found in Finland) as has
been said pulling up the flap is favourite (it certainly can't do any
harm) The prob was fixed on the RAF standard of jet by removing
an inboard section of the flap vane (a type of slotted flap was
used) This put up the approach config stalling speed a tad but

Age: 77
Posts: 1,216

there was a lot in hand on the RAF spec so no sweat.


Later when more lift was needed on heavy external stores
overseas versions and especially the USN T-45 some strakes fixed
to the fuse just below and near the LE of the tailplane (think of
them as huge VGs if you will) reduced the local AoA of the
tailplane and fixed the problem allowing the restoration of a full
span flap vane.

29th November 2004,


00:32

#10 (permalink)

blackmail

hello every one

Join Date: May 2003


Location: france
Posts: 179

the horizontal stabilizer is basically a wing flying upside down with


the lift vector pointing downwards, assuming the cg is forward of
the center of pressure. so in case of tailplane stall you have to use
opposite techniques to recover: gently pull on the yoke & slowly
add thrust. some accidents happened when selecting full flaps
with a violent & uncontrollable pitch down due to tailplane icing.
viscounts had this problem as stated in another post. i guess that
if you change configuration & the aeroplane has an unexpected
reaction, immediately go back to the previous flap setting & use
above mention recovery techniques if tailicing is suspected. read
an article about this long ago, where this was flighttested with a
twinotter.(scary!). pitchanomalies & unusual vibrations in icing
conditions all can point to tailicing.
always wondered why b737 has no tail deice/antiice protection
devices. boeing says it is not needed, well?

29th November 2004,


01:21

oldebloke
Join Date: Apr 2004
Location: vancouver
oldebloke
Posts: 261

29th November 2004,


01:25

Astra driver
Join Date: Oct 2004
Location: Los Angeles

#11 (permalink)

Cases in point: DC4 Slick Airways,10mar64.Boston-"Loss of


balancing forces on stabalizer, due ice accretion,causing aircraft to
pitch nose down too low to effect recovery..
Viscount,Skyline sweden,15jan'77-"ice on leading edge of
Stabalizer causing flow seperation and Stabalizer stall..
Viscount,Capital airlines06Apr58,Michigan,Undetected ice
accretion on stabalizer,in conjunction with specific speed and
configuration,caused loss of pitch control...

#12 (permalink)

We are usually taught tail stall recovery at every other recurrent


session while attending Flight Safety at ILG. (Wilmington, DE
USA)

Age: 47
Posts: 154

We are told that a tail (Icing) stall will usually occur at higher
airspeeds while deploying flaps as opposed to the wing stall,
which would, of course be at lower speeds and in any
configuration.
The stall, when it occurs is identical to wing stall, the only clue to
the diffference being the circumstances under which they occur.
The recovery procedure is to apply max power and pull back on
the yoke while returning the flaps to their previous setting.
I think that if you are in potential icing conditions and prime
yourself for the possibility of a tail stall you should be able to
recognise the onset before it becomes a problem.

29th November 2004,


01:36

411A
Join Date: Mar 2000
Location: Arizona USA
Posts: 7,508

#13 (permalink)

Oldebloke was kind enough to provide the references needed, so it


sure does seem to be a problem on some particular types.
Similiarly, as blackmail mentioned with the B737, the Lockheed
TriStar does not have tail anti-icing either.
Altho the TriStar is a unique design amongst civil jet transports with
its 'all flying' stabilizer, I would suspect many jets do not need tail
anti-icing due to the fact that the horizontal stab moves over quite
a large arc, due to aircraft trim requirements.

29th November 2004,


12:49

PickyPerkins
Join Date: Jul 2002
Location: 40N, 80W
Posts: 169

#14 (permalink)

Milt said:
Quote:

Incidently FBW and artificial stability will be a way we can design


for a zero tail load or even an up load on the tail for cruising
flight..It's happening with military aircraft and close for nonmilitary using artificial stability. But the elevators will then have a
very busy time preventing runaway in pitch.
Just curious about how an inherently unstable aircraft such as the
F117 reacts to icing? Since it relies on multiple computers to fly at all,
does icing make any difference (short of blocking the engine intakes
and freezing the controls in place). Does it have de-icing equipment?
This web-site
"http://www.grc.nasa.gov/WWW/RT1998/5000/5840ratvasky.html"
describes a NASA/FAA Tailplane Icing Program (TIP), and mentions

that it was initiated because of at least 16 accidents resulting in 139


fatalities attributed to such stalls. The objectives of this program
were to improve understanding of iced tailplane aeroperformance and
aircraft aerodynamics and to develop training aids and design tools to
expand the awareness of ice-contaminated tailplane stall (ICTS). A
23-min video, "Tailplane Icing", was produced.
FAA Advisory Circular AC No: 23.143-1 "Ice Contaminated Tailplane
Stall (ICTS) Date: 12/20/01" contains several warnings about the
hazards of testing aircraft with stalled tailplanes, e.g.
Quote:

WARNING
Some hazard is associated with ICTS flight testing. It is important
that the applicant take appropriate precautions in the conduct of
these tests and the flight test crew very carefully considers risk
mitigation that includes defining minimum altitudes, conducting
tests in a build-up manner, and providing emergency escape and
parachute provisions.
Cheers,
Last edited by PickyPerkins : 29th November 2004 at 19:16.

29th November 2004, 16:37

alf5071h
Join Date: Jul 2003
Location: An Island Province
Posts: 697

#15
(permalink)

Re does
seem to be a
problem on
some
particular
types.
The
mechanism of
tail stall is
very
dependant on
the type of
aircraft
control
system and of
course the
formation of
ice (design
and operation
of the anti /
de icing
system).
I understand
that it is rare
for the tail to

be fully
stalled; the
dominant
feature in an
icing 'tail stall'
is a change in
pitching
moment as
the tail force
reduces as
explained
above. In
aircraft with
manual
controls, the
pilot feels the
change in
control force
on the
elevator
associated
with the
pitching
moment and
in extreme
cases, the
control
column is
forced
forward,
hence the
need for a
hard pull
back.
In aircraft
with powered
controls, the
pilot does not
sense the
actual control
force, nor can
the control
column be
moved by the
surface
(irreversible
system).
In aircraft
with effective
anti icing
systems (little

or no ice
accretion),
there is little
or no change
in pitching
moment; or
where there
is a small
change in
pitch it can be
trimmed out.
NB in large
aircraft with
trimming tails
the effect is
negligible and
I suspect this
may be a
reason for not
requiring anti
icing systems
on the
tailplane.
The main risk
of an icing
induced tail
stall (as with
any icing
hazard)
occurs in
severe icing
where the
anti / de icing
system is
overcome by
the
accumulating
ice (definition
of severe
icing
aircraft type
dependant).
The crew are
responsible
for identifying
and exiting
the severe
conditions as
aircraft are
not
certificated

for such
operations.
Accidents /
incidents
have occurred
where crew
continue with
an approach
in unsuitable
conditions (or
with an icing
system
failure),
maintain a
higher than
normal
airspeed
often required
for main wing
stall
protection,
and then
lower flap
which
increases
susceptibility
to 'tail stall'.
The modern
industry is
somewhat
dismissive of
the hazards
of ice. .
Please see
and use A
Pilot's Guide
to In-Flight
Icing.
If an aircraft
is shaking it is
either too
fast, too slow,
or covered in
ice.
In aviation
there are
three types of
ice. Good Ice,
Bad Ice and
Hazardous

Ice.
Good Ice is
found in the
galley.

29th November 2004, 21:53

ehwatezedoing
Drain Bamaged
Join Date: Mar 2000
Location: Canada
Age: 42
Posts: 209

#16
(permalink)

Had a Flight
Safety
seminar
regarding
icing, part of
it was about
tail stall.
All infos were
comming
from Nasa
Lewis
research
center (as
mentionned
by
PickyPerkins).
Now, back to
my notes....
A common
design lead to
this kind of
problems:
Turbo props fall
nicelly into this
category!

-Unpowered
flight
controls,
-Large flaps
deflection,
-Deicing
boots,
-Sharper
horizontal tail
than wings.
3 paths lead
to tail stall
conditions if
there is ice in
the tail:

-Flaps,
-Speed,
-Power.
There is
usually no
clue about it
until
configuration
change.
First thing is
of course
correctly
diagnost the
problem:

You will miss those


under auto pilot.
And many time
those are flaps full
extention
symptoms

-Lightening of
the control,
stick
lightening in
the forward
direction,
-Difficulty
trimming the
A/C,
-Onset of
pilot induced
oscillations,
(worse case
scenario
where the
yoke will snap
forward)
-Buffeting in
controls not
the airframe.
(You must
differentiate
airframe
buffet to yoke
buffet)
To recover
from a tail
stall:
-Pull back on

yoke,
-reduce flaps,
-reduce
power.
(maybe
aircraft
specific)
More
simple--->
The universal
tail stall
recovering:
-Yoke back,
-Flaps to
previous
setting.
Immediate return for landing

Just want to get some feedback on the above issue under the following set
of circumstances :
You take off at MTOW in your 777 or any other two engined ETOPS aircraft
capable of fuel jettison. Shortly after take off you have an engine failure.
Under the rules of two engined flight this is an emergancy and requires a
landing at the nearest suitable airfield.
The question therefore is do you
A) In a mannered and disciplined fashion return for a landing at the earliest
time and not waste time jettisoning fuel believing that an overwieght landing
is a lot better than being in the air any longer with an engine inoperative.
or
B) Take the time to go and jettison fuel knowing that the other engine has
been designed to run for 120-180mins so taking time for fuel jettison would
not be a significant risk and would reduce the chances of an incident on
landing.
Also in additon to this would your attitude change depending on what type of
engine failure had occured e.g. a simple rundown vs. a surging engine.

8th December 2004, 03:42

Capt Fathom
Join Date: Jun 2001
Location: FNQ
Posts: 1,497

#2
(permalink)

Answer (A) is
correct.

#3 (permalink)

8th December 2004, 07:21

411A

Like nearly everything in aviation, it depends...

Join Date: Mar 2000


Location: Arizona USA
Posts: 7,508

Well, in my view, answer "B' would be the preferred choice,


depending on the nature of the engine failure.
OTOH, I would not want to steam around for two hours either.
I would expect to follow the particular airlines' standard
procedures, normally. barring any unusual circumstances.
Having said all this, nothing wrong with an overweight landing
either, if called for for any reason whatsoever.
SR111 comes to mind.

#4 (permalink)

8th December 2004, 09:23

Right Way Up
Join Date: Jan 2001
Location: home
Posts: 788

Quite a lot of variables in this situation. Runway length, weather


being a few! For an unextinguishable engine fire the scenario is
black and white, for an engine failure it really is down to the
commanders discretion. A factor to consider are where can I
dump fuel. If I have to fly 100 nm to dump fuel then no, if I can
dump fuel in the locality of an airfield then why not. I think 411a
has it right, there is no particular rush but also there is normally
no problem landing overweight. Personally I do not like to fly
away from an airfield, one possible reason for the engine failure
could be poor maintenance, 411a will probably expand more, but
I remember an L1011 (poss EAL), losing engine after engine on
the way to the Bahamas, and only just landing at MIA after type
of dog-up with oil seals. Also a Queens flight 146 doing similar
and landing with virtually no power.

#5 (permalink)

8th December 2004, 11:23

Dehavillanddriver

As 411 and others have said - it depends.

Join Date: Jan 1999


Location: Brisbane
Posts: 534

If it was burning I'd come straight back


If it was a straight cut - ie the engine ran down - I'd dump
As for altitude, I'd stay above the 25 nm MSA (and within 25nm)
given that the charts (in my country at least) have a 25 nm MSA
published on the Jepps.
Is a straight cut an emergency or an urgency situation - would

you call mayday or pan?


Unless it was burning (or had damaged the aircraft during its
death) I reckon it is an urgency situation rather than an
emergency - but there are many differing views on this point.

#6 (permalink)

8th December 2004, 13:28

VR-HDB
Join Date: Oct 2004
Location: Europe
Posts: 31

Quote:

If it was a straight cut - ie the engine ran down - I'd dump


Quote:

Is a straight cut an emergency or an urgency situation - would


you call mayday or pan?
At first I though it seemed a bit contradictory to call mayday, and
then take the time to go out dumping fuel. But, the fuel dump is
(primarily) done to ensure sufficient performance for a possible G/A
(appch climb) - so perhaps the surrounding terrain clearance should
also be considered.
So maybe the situation is considered to be critical enough for a
mayday - but the crew still decides to dump fuel to increase
performance.
8th December 2004, 14:21

DBate
Join Date: Jan 2003
Location: Europe
Posts: 210

#7 (permalink)

Since we are looking at a scenario flying a two-engined aircraft, I


would always decide on a mayday call.
Rember, you have only one engine left, so there's no redundancy.
Concerning 411As comment about SR111: Sad but true, even if
the crew had immediately decided to do an overweight landing an
prepared the approach, they would'nt have made it.

8th December 2004, 17:26

6100
Join Date: Jun 2004
Location: Idaho
Posts: 60

#8 (permalink)

In a two engine aircraft with one engine failed, there is only one
reason not to come back and land asap (with appropriate
checklists and approach preparation complete) and that is
performance (landing performance or go around performance).
Even then you can decide to waive those requirements if for
instance you have a hugely long runway, cavok, and no obstacles.
I look at it this way. When you are responsible for passengers
lives, be it one or 400, then you must always have a contingency

plan. If you have already had one engine fail, then what is your
contingency plan if you are away from the airfield dumping fuel
and the other engine fails?
You are in a fair bit of trouble I would say.
It is not logical to apply the ETOPS theory. The only reason we fly
for 3 hrs on one engine in the ETOPS case is because we have to.
We would never overfly a suitable airfield on one engine just to
get to the designated ETOPS alternate.
That's my two bobs worth anyway

#9 (permalink)

8th December 2004, 18:23

Doors to Automatic
Join Date: Dec 1999
Location: EGNX
Posts: 670

What would the position be at an airport with a relatively short


runway (e.g. a 777 at Birmingham).
Would the procdedure be to land immediately so that time flying
on one engine is minimised or divert to an airport with a longer
runway which is safer for an overweight landing?

#10 (permalink)

9th December 2004, 01:16

oldebloke
Join Date: Apr 2004
Location: vancouver
oldebloke
Posts: 261

B74,your heading was an immediate return to landing?,and then


you pose an 'engineout'as the concern!!As has been covered ,do
the drills.checklists,and 'briefings'all in a pedantic manner,land on
the longest if no fuel dump etc....How about a more expedient
question in the case oh a PAX heart attack,or the second 'bottle'
light' doesn't extinguish??

#11 (permalink)

9th December 2004, 04:43

mutt

oldebloke,

Join Date: Sep 1999


Location: ME
Posts: 3,622

Landing performance is governed by three restrictions:


[list=1][*]Approach Climb/landing climb.[*]Field Length Limit
weight.[*]Certificated Landing weight.[/list=1]
By regulation an airliner must be capable of meeting the approach
climb and landing climb requirements 15 minutes after takeoff,
with or without a fuel jettison system.
A quick check of a field length limit chart will give you the weight
limitation for Nbr2.

In an emergency, the certificated landing weight can be exceeded,


up to the limits of Nbr 1&2.
Therefore, once the crew has established that they can safely land
on that runway, its their decision as to the urgency. For your
example of PAX heart attack or fire light, the aircraft will be
capable of landing after 15 minutes, based upon a MTOW
departure.
Mutt.

#12 (permalink)

9th December 2004, 04:58

Dehavillanddriver
Join Date: Jan 1999
Location: Brisbane
Posts: 534

I guess that the question is what constitutes an immediate return


to land?
Do you mean a quick circuit to get the aircraft back onto the
ground?
or do you mean something else.
let me preface the below by saying that each case is individual
and I am talking in general terms here!
I brief that if we need an emergency return we will do a LH or RH
circuit (as applicable) , nor a non emergency return we will go and
climb up to the 25 nm msa, sort it out and then come back when
we have our poo in a pile.
I reckon an emergency return is when the machine is burning, or
has some sort of handling problem.
If it was a rundown or similar I'd climb up to the MSA do the
checklists, brief the cabin crew and pax, brief the approach etc
(not necessarily in that order for the pedants)
I reckon a major cause of stuff ups (in the sim at least) is people
rushing and forgetting or not considering something that they
would never normally dream of forgetting.
Taking the time to sort everything out, run all the appropriate
checklists, speak to the required people, discuss amongst
yourselves (Capt and F/O) the options, have we forgotten
anything - any suggestions etc, is a very worthwhile thing to do in
my opinion.
In my mind this is where I would make the distinction between a
PAN and a MAYDAY.
The statistical probability of having a failure, followed by a totally
unrelated failure of the other engine is incredibly remote - note

the word unrelated...so much so that I reckon that the risk


associated with taking the extra time to dump, do all the other
things etc is outweighed by the risk of stuffing it up if you rush.
No matter what you decide on the day, the arm chair
quarterbacks will come up with a reason why you did it incorrectly
anyway - so why worry!

#13 (permalink)

9th December 2004, 19:59

oldebloke
Join Date: Apr 2004
Location: vancouver
oldebloke
Posts: 261

Mutt,sorry but you missed the point of an 'Immediate return to


landing'...In the case presented one had an engine out-No
requirement for an Immediate landing..I changed the scenario to
develope thoughts on where one would 'abreviate' the checks to
get on the ground ASAP......
On the 727 there was no concern-leave the bugs as they
were,land O'weight with reduced flap( to meet the Climb
gradient)..
Same for the 320,activate the 'approach page' to get the
'proper'speeds,land with flaps 3 (again to meet the Climb
gradient)use longest runway..
These comply with an Immediate return to land..
Cheers(food for thought whilst in cruise or walking the dog)

10th December 2004,


01:36

#14 (permalink)

DFC

Yak, Yak, Yak............!!!

Join Date: Mar 2002


Location: Euroland
Posts: 2,586

Anyone here ever flown a glider?


Stay withn gliding distance and who cares if you have power or
not.
In the UK is it perfectly OK to dump fuel at 10,080+ft above egll
and lower if you declare an emergency!!!!
So, having departed EGLL, why not orbit in the overhead where
there is no traffic.
Anyone here been vectored through the egll overhead on a regular
basisinbound to egll?
ATCO's avoiding red lines keep quiet!!
1 Eng out off egll. I start the dump ASAP........my requirement to
save life always resides above the fish life in the waste lakes.
Regards,

DFC
PS,
One can only use ETOPS in circumstances where ETOPS is
required. ...............Who in their right mind would hold close to a
suitable landing place with 450 pax on one engine?
Unless wx dictated otherwise........which brings us to the
departure alternate!
regards again,
DFC

The purpose of a leading edge droop?

answer:
to increase wing camber, and delay separation of the
airflow when trailing edge flaps are lowered.
Can anyone try to explain me, what a leading edge droop
is? I would be very grateful.

24th November 2004,


17:24

Tinstaafl
Join Date: Dec 1998
Location: Escapee from
Ultima Thule
Posts: 2,930

24th November 2004,


17:50

Bally Heck
Join Date: Oct 1999
Location: United Kingdom
Posts: 386

#2 (permalink)

An analogy using your forearm & hand to represent a cross section


through the wing ie leading edge to trailing edge:
Hold your forearm horizontally in front of your eyes. Your hand
represents the leading edge of the 'wing'. Relax your wrist. That's
LE droop.

#3 (permalink)

Tinstaafl's analogy is more convincing if the other hand is placed


on your hip and you pout your lips.

24th November 2004,


18:33

gas path
Usual disclaimers apply!

#4 (permalink)

Leading edge droop is a transonic wing thing isn't it to improve


low speed lift? As opposed to leading edge flaps and slats etc.
Droop being fixed as opposed slats etc that move.

Join Date: Nov 1999


Location: windsor
Posts: 660

gas path

Did Hawker Siddleys finest, leading edge device stay sealed to the
wing through all phases of it's operation?

Usual disclaimers apply!


Join Date: Nov 1999
Location: windsor
Posts: 660

25th November 2004,

#7 (permalink)

06:20

nilnotedtks

...Or put simply, a droop is a slat without a slot !

Join Date: Aug 2000


Location: Dubai, United
Arab Emirates
Posts: 65

25th November 2004,


16:12

#8 (permalink)

Astra driver

Odd one,

Join Date: Oct 2004


Location: Los Angeles
Age: 47
Posts: 154

Thanks for that link, I learned a couple of new things from it.

25th November 2004,


18:00

PAXboy
Paxing All Over The World
Join Date: May 2001
Location: Hertfordshire, UK.
Posts: 4,628

26th November 2004,


09:52

subsidence

AD

#9 (permalink)

(non-pilot) As I recall, the Vickers VC-10 had leading edge droops


as well? Were those droops or slats or slips and slops? (Can't get
all of this fly-thingummy-jargon)
-------------------"I tell you, we are here on Earth to fart around, and don't let
anybody tell you any different." Kurt Vonnegut, Jr.

#10 (permalink)

Thank you very much for your replies, that was very helpful!

Join Date: Mar 2004


Location: Miami
Posts: 33

26th November 2004,


11:20

keithl
Join Date: Feb 2003
Location: Scotland
Age: 63
Posts: 429

#11 (permalink)

...especially the K. Vonnegut quote, PAXboy. Did he really say


that? It fits rather well with my own philosophy.

29th November 2004,


11:39

#12 (permalink)

blackmail

hello everyone,

Join Date: May 2003


Location: france
Posts: 179

leadingedge droop & krugerflaps have the same aim : lift


augmentation by increased camber allowing for lower stallspeeds
& thus lower approach speeds. both systems have advantages &
disadvantages.
krugerflap : advantage : it's retractable, so you get rid of the
extra drag in cruise flight.
disadvantage : relatively complex mechanical retraction/extension
system & extra weight.
leading edge droop : opposite : mechanical simple & permanent,
but extra drag in cruise.
both systems have their over/underpressure zones modified,
compared to a normal cambered profile without kruger or le
droop.
also the impactpoint(v=0), moves from intrados(wingunderside)
to extrados(wingupperside).
eg: b737 has krugerflaps on the inboard leading edges, the
venerable french caravelle had leading edge droop.
Last edited by blackmail : 29th November 2004 at 11:49.

29th November 2004,


16:42

PAXboy
Paxing All Over The World
Join Date: May 2001
Location: Hertfordshire, UK.
Posts: 4,628

1st December 2004, 10:03

OVERTALK
Join Date: Dec 1998
Location: England
Posts: 236

#13 (permalink)

keithl: This is the attribution that I have:

"And I tell you, we are here on earth to fart around,


and don't let anybody tell you any different."
(Kurt Vonnegut Interview in Inc. Technology January 1996,
Vol.17, No. 17)

#14 (permalink)
Challengers and CRJ's and lack of Leading Edge Devices

http://www.tc.gc.ca/tdc/publication/...100/14180e.pdf
If anybody wants to worry about RJ's and CRJ's and Challenger
wings and lack of LE devices, read that link.
For the knowledgeable it will send chills up your spine. Challenger

pilots should be very careful about icing, de-icing and rotating at


too slow an airspeed ( or too rapidly ).
With the sort of wing-sweep the CL600 has, it should have been
given a leading edge device (same with CRJ's).

1st December 2004, 12:04

Mad (Flt) Scientist


Join Date: Sep 2002
Location: La Belle Province
Posts: 1,519

2nd December 2004, 00:49

Elliot Moose
Join Date: Feb 2000
Location: Montreal
Posts: 116

#15 (permalink)

Since that report discusses theoretical work on two nominal


sections and CFD work on the Fokker F28, I'm not sure I 'get' the
link to Canadair products.

#16 (permalink)

There are around 1600 CRJ 100/200 and Challenger aircraft in


service today (don't count the 700/900 series as they have slats)
and until VERY recently, everybody flew them with no problems
through all types of weather.
That said, all decently trained pilots of these aircraft are made
well aware that the lack of leading edge devices combined with a
supercritical airfoil means that the airfoil will brook very little
contamination without serious consequenses . It is actually a
maintenance function to polish the leading edges of these aircraft
at designated intervals as flying with them unpolished will affect
the performance, so obviously even a little bit of ice buildup
before takeoff will cause grief if not dealt with. In flight, however,
there is negligible effect as the anti-ice system works well, and
there is seldom need for extended flight in significant ice in jet
aircraft anyways.
At the time that both of these series of aircraft were designed,
Canadair was basically called foolish. Who would buy a wide-body
business jet when sleek lears and citations were the norm? 500600 airframes later we see it to have been a good idea. The CRJ
was an even wilder design idea. Nobody bought the idea that a jet
could be fiscally viable with only 50 seats. The 1000th of the 50
seater will soon come off the line. My point here is that the cost of
adding the complexity of slats to these aircraft was not prudent at
the time that these were being designed. It is a bit of a pain to
have to respect the limitations of the airfoil, but that is about it.
Full stalls (to the pusher) as we do in production test flights are
definitely a no-no and the almost complete lack of pre stall buffet
is not an exaggeration. Not to be done except under very
controlled conditions! By the time the 700 was designed, almost
1000 of the 50 seaters had been ordered and it made sense to go
the extra distance and add slats. Yes circling approaches are a lot
easier in the 700 with its slats and lower speeds, but they are

certainly not dangerous in a 200 if proper procedures are


followed.
By the way, Odd One, Thanks for the neat link!
Some
absolutely fantastic nerd material there, which I will definitely
spend some time perusing!
(EM dons his anorak and puts
fresh cello tape on the glasses.............)

Você também pode gostar